0% found this document useful (0 votes)
7 views440 pages

Final Exam s6g3_compressed

The document presents a series of clinical cases involving patients with various forms of chest trauma, respiratory distress, and associated symptoms, along with questions regarding diagnosis and management. It includes scenarios of tension pneumothorax, lung abscess, and bronchogenic carcinoma, as well as discussions on chest tube management and surgical considerations. Additionally, it covers anatomical questions related to pulmonary arterial anatomy and complications of surgical procedures.

Uploaded by

zlozlo2417
Copyright
© © All Rights Reserved
We take content rights seriously. If you suspect this is your content, claim it here.
Available Formats
Download as PDF, TXT or read online on Scribd
0% found this document useful (0 votes)
7 views440 pages

Final Exam s6g3_compressed

The document presents a series of clinical cases involving patients with various forms of chest trauma, respiratory distress, and associated symptoms, along with questions regarding diagnosis and management. It includes scenarios of tension pneumothorax, lung abscess, and bronchogenic carcinoma, as well as discussions on chest tube management and surgical considerations. Additionally, it covers anatomical questions related to pulmonary arterial anatomy and complications of surgical procedures.

Uploaded by

zlozlo2417
Copyright
© © All Rights Reserved
We take content rights seriously. If you suspect this is your content, claim it here.
Available Formats
Download as PDF, TXT or read online on Scribd
You are on page 1/ 440

than you

k
Re
v iew
Are you
ready?
LETS
STAR
T
• Male patient 25 years old presented to emergency department by shortness of
breathing and chest pain after motor car accident and direct chest trauma 6
hours ago

• On examination the patient is dyspneic cyanosed hypotensive and tachycardic

• Resonant on percussion with absent air entry on the right side of the chest.

• What is:
What is your next step?.
Radiological CXR findings
Surgical management
Female patient 20 years old with MCA (or FFH) with chest trauma
6hours ago. patient has chest pain, shoring of breathing, cyanosed.
Absent air entry on Lt side, sweety hypotensive BP 90/60mm/Hg, pulse
110beat/min
• What is your diagnosis

• What is your next step?.

male patient 45 years old presented by stab chest at Lt 5th intercostal


space parasternal line. patient shocked, pale; equal air entry, distended
neck veins BP 80/50mm/Hg, weak Pulse 136 beat/min
• What is your diagnosis?

• What is your next step?.


Male patient 19 years old with gun shoot to the right chest inlet and .
outlet. The patient is pallor, dyspneic, sweety, hypotensive, diminished
air entry on Rt side, BP 90/60mm/Hg, pulse 114beat/min
• What is your diagnosis?

• What is your next step?.

Male patient 30 years old presented by gradual shorting of breathing


and git disturbance with past history of motor car accident under going
urgent splenectomy on examination there is diminished air entry on it
side
• What is your diagnosis

• What is your next step?.


Female patient 50 years old with past history of cancer
breast have chemo radio therapy presented by sever
shorting of breathing orthopnea pallor hypotensive
muffled heart sound with equal air entry on both sides
• what is your diagnosis
• What is your next step?.
Male patient 60 years old smoker (30 cig/day), mine worker
presented with chronic cough >3 weeks not respond to medical
treatment, recently coughing of blood, chest pain at right arm, with
ptosis of right eye, headache, attacks of fainting, shortness of breath,
back pain, history of loss of weight and loss of appetite, hoarseness of
voice,.

• Ex.:- Cachectic, edema and swelling of right upper limb and right side
of the face, ptosis-myosis -anhydrosis of Rt eye,
• By auscultation decreased air entry on RT side of chest.

• What is your next step?.


• Male patient 40 years old, smoker, uncontrolled DM with
recurrent diabetic coma, fever, coughing of huge amount of
purulent sputum when laying forward or laying on his left side,
shortness of breath, easy fatigability, anorexia, chest pain on
right side, swelling and tenderness at the right lateral chest wall,
severely pallor (toxic), lower limb edema, cachexia, decreased
air entry on the right side and clubbing of fingers

• What is yourdiagnosis andmanagement ?


• Early management of tension pneumothorax
• -chest tube -needle decompression

• -surgical treatment - follow up

• Definitive symptoms of suppurative lung disease


• -chronic cough for more than 3 weeks -high fever specially at night

• -expectoration of hug amount of purulent sputum- -chest pain

• Radiological pictures of lung Abscess


• -cavity with air fluid level -mediastinal shift

• -jet black opacity -elevated copula of diaphragm


which of the following lobes has the most variable
pulmonary arterial anatomy ?
A. Right upper lobe
B. Right middle lobe
C. Right lower lobe
D. Left upper lobe
The most common complication of surgical ligation of PDA
is :
• Select Best Answer
A. Phrenic nerve injury
B. Recannalation of the ductus arteriosus
C. Chylothorax
D. Left recurrent laryngeal nerve injury
E. Cerebrovascular accident
•Which of the following beast describes the most
common site of defect in Morgagni's Hernia ?
• Select Best Answer
A.Anterior , to the left
B.Anterior , to the right
C.Posterior , to the left
D.Posterior , to the right

•The azygos vein separates which of the two lymph


node stations ?
A.2R and 4R
B.4R and 10R
C.10R and 11R
D.4R and 4L
• Which of the following incisions is the best approach for most upper
tracheal lesions:
• Select Best Answer
A. Right Thoracotomy
B. Left Thoracotomy
C. Sternotomy
D. Cervical Collar incision
• Which of the following is an Indications
ofchest tube insertion?
• Huge lung cyst with mediastinal shift
• Complete consolidation collapse of the lung
• Huge lung abscess with air fluid level
• Pneumomediastinum
• Hydropneumothorax
Which is the Safest site forchest tube
insertion (usually)?
• 2nd intercostal space midclavicular line
• 5th intercostal space midclavicular line
• 2nd intercostal space midaxillary line
• 5th intercostal space midaxillary line
•Which one of the following is a complications ofchest
tube insertion? injury to :-
• Heart
• Lung
• Liver
• Stomach
• Esophagus

•Which one of the following is a Contraindication of


chest tube insertion ?
• Multiple fracture ribs
• Internal hemorrhage
• Lung tear
• Coagulopathy
What is your action if the chest tube not
functioning?
• Follow up
• Remove the tube
• CXR
• Suction the tube
When chest tube removed?
• After 5 days
• After 10 days
• After complete lung inflation
• When discharge became < 100ml/day
Management of Chest Tubes: Common
Questions after Placement
How should the chest tube be monitored
each day?
 Drain output
 Swinging
 Air leak
 Dressing change
 Signs of infection or hematoma at drain site
 Symptoms-SOB, cough, chest pain
Management of Chest Tubes: Common
Questions after Placement
What has happened if no swinging is
observed in the chest drainage system?
 Displaced or dislodged chest tube
 Kinked tubing
 Malpositioned chest tube
 Tube blockage
Management of Chest Tubes: Common
Questions after Placement
When should suction be applied?
 Large air leak
 Subcutaneous emphysema
 Failure of lung to re-expand on x ray
When to operate :-
•1-Pneumothorax
•2-Hemothorax
When to remove Chest Tube
• Lung fully expanded: clinically and radiologically
• No air leak > 24 hrs
• No fresh/altered blood/pus
• drainage < 100 cc
Removal of Chest Tube
•Chest radiogram is obtained

•If no re-accumulation of air or fluid, chest tube


can be removed
Post Chest Tube removal
pneumothorax
•< 1/3rd lung parenchyma
• patient not in distress
@Observe
@Repeat Chest X-ray
= Act as necessary
• Male patient 40 years old nonsmoker uncontrolled DM with
recurrent coma, fever, coughing of purulent sputum when laying
forward or laying on his right side, chest pain on left side,
shortness of breath, easy fatigability, anorexia, swelling and
tenderness at the lateral left chest wall, severe pallor (toxic),
lower limb edema, cachexia, decreased air entry on the left side
and clubbing of fingers

• (Suppurative lung disease)


• Male patient 60 years old smoker (30 cig/day), mine worker presented
with chronic cough >3 weeks not respond to medical treatment,
coughing of blood, chest pain with right arm with ptosis of right eye,
headache, attacks of fainting, shortness of breath, back pain, history of
loss of weight and loss of appetite, hoarseness of voice, cachexia.
• by auscultation: decreased air entry on RT side apical zone, edema and
swelling of right upper limb and right side of the face, ptosis –myosis -
anhydrosis of Rt eye
• (Bronchogenic carcinoma)
• Which of the following incisions is the best approach for most upper
tracheal lesions:
• Select Best Answer

A.Right Thoracotomy
B.Left Thoracotomy
C.Sternotomy
D.Cervical Collar incision
• Which of the following beast describes the most common site of
defect in Morgagni's Hernia ?
• Select Best Answer
A.Anterior , to the left
B.Anterior , to the right
C.Posterior , to the left
D.Posterior , to the right

• The azygos vein separates which of the two lymph node stations ?
A.2R and 4R
B.4R and 10R
C.10R and 11R
D.4R and 4L
• which of the following lobes has the most variable pulmonary arterial anatomy ?
A.Right upper lobe
B.Right middle lobe
C.Right lower lobe
D.Left upper lobe
• The most common complication of surgical ligation of PDA is :Select Best
Answer
A.Phrenic nerve injury
B.Recannalation of the ductus arteriosus
C.Chylothorax
D.Left recurrent laryngeal nerve injury
E.Cerebrovascular accident
• The diaphragm is the primary muscle of inspiration, contracting downward to
increase intrathoracic volume. The diaphragm
• A. has crural fibers that connect to the ribs.

• B. has aortic hiatus at T10 level.

• C. contains oesophageal hiatus that transmits branches of the right phrenic nerve.

• D. has foramen of Morgagni that transmits superior epigastric branches from internal mammary
artery.

• E. has caval hiatus at T12 level.

• Answer: D
• The thoracic inlet is essentially a hole surrounded by a bony ring, through which
several vital structures pass. Which one of the following structures does not
pass through the thoracic inlet?
• A. oesophagus

• B. trachea

• C. brachial plexus

• D. left recurrent laryngeal nerve

• E. right internal jugular vein

• Answer: C
• The mitral apparatus is composed of the left atrial wall, the annulus, the leaflets, the chordae tendineae, the papillary
muscles, and the left ventricular wall. Which one of the following statements regarding the mitral valve is correct?

• A. The mitral valve is located obliquely anterior to the aortic valve.

• B. The left atrial myocardium extends over the proximal portion of the anterior leaflet.

• C. The mitral annulus is a continuous ring around the mitral orifice.

• D. The mitral annulus is a fibromuscular ring that connects with the leaflets.

• E. The straight border of the annulus is posterior to the aortic valve.

• Answer: E

• The most anterior cardiac chamber is

• a) Right ventricle

• b) Left ventricle

• c) Right atrium

• d) Left atrium
• Pulmonary sequestration is an abnormal segment of lung tissue that has no communication with
the tracheobronchial tree. Pulmonary sequestration

• A. usually receives its arterial blood supply through the hilum.

• B. predominantly affects females.

• C. in an extralobar location is the predominant pathology.

• D. venous return is usually in systemic venous system.

• E. in an extralobar location has its own visceral pleura.


• An SHO is about to perform her first pleural fluid aspiration. If you
are supervising her where would you ask her to insert the aspiration
needle to avoid injuring lung or neurovascular elements?
• A. the top of interspace 8 in the midclavicular line

• B. the bottom of interspace 8 in the midclavicular line

• C. the top of interspace 8 in the midaxillary line

• D. the bottom of interspace 8 in the midaxillary line

• E. the top of interspace 8 in the scapular line


• Thoracic outlet syndrome (TOS) is defined as a constellation of symptoms related to compression of the neurovascular
elements found in the thoracic outlet. TOS

• A. has 4 types.

• B. in 90% of all cases progresses to Paget-Schroetter syndrome.

• C. is more common in patients 20 to 40 years of age.

• D. is more common in men than women.

• E. symptoms are elicited by Allen test.

• Pancoast tumours are located in the apex of the lung and involve through tissue contiguity the apical chest wall and/or the
structures of the thoracic inlet. Which one of the following statements regarding Pancoast tumours is correct?

• A. Pancoast tumours account for 25% of all bronchogenic carcinomas.

• B. Squamous cell carcinomas account for 2/3 of all Pancoast tumours.

• C. Pancoast tumours are by definition T2 tumours.

• D. Induction chemo-radiotherapy is the standard of care for any potentially resectable Pancoast tumour. E. Surgery for
Pancoast tumours is associated with 50% mortality rate.


•The mitral apparatus is composed of the left atrial wall, the annulus, the leaflets, the chordae tendineae,
the papillary muscles, and the left ventricular wall. Which one of the following statements regarding the
mitral valve is correct?
•A. The mitral valve is located obliquely anterior to the aortic valve.
•B. The left atrial myocardium extends over the proximal portion of the anterior leaflet.
•C. The mitral annulus is a continuous ring around the mitral orifice.
•D. The mitral annulus is a fibromuscular ring that connects with the leaflets.
•E. The straight border of the annulus is posterior to the aortic valve.
•A 24-year-old man is admitted as a trauma call to the emergency department having sustained a stab
wound to the anterior chest. He is haemodynamically compromised with a systolic blood pressure of 50
mmHg, heart rate of 130/min and elevated jugular venous pressure. Upon inspection there is a 2 cm
laceration left to the sternum at its lower third. Which one of the following statements is correct?
• A. Pericardiocentesis is a reliable diagnostic tool for penetrating cardiac trauma.
•B. Upon emergency thoracotomy, a laceration to the ventricle should be repaired with continuous 4-0
prolene.
•C. Repair of ventricular wounds without CPB can be facilitated with up to 10 minutes of inflow occlusion
after placement of caval clamps.
•D. Ventricular injuries are more common than atrial injuries.
•E. Repair of lacerations near coronary vessels will inevitably lead to severe compromise of their flow.
• On opening the right atrium, you see large amounts of blood coming
via the coronary sinus, which is also very enlarged. You suspect
• A. partial anomalus pulmonary venous drainage
• B. left superior vena cava
• C. unroofed coronary sinus
• D. unrecognised ASD
• E. inadequate drainage from bicaval cannulation

• A young female with a long term intravenous jugular catheter is waiting for surgery. Two days later, she presents with a cold left
leg. The best test for this patient now is

• A. Femoral angiogram

• B. Cardiac catheterization

• C. Echocardiogram

• D. Septic screen

• E. Venogram

• A 3 years old boy undergoes ligation of patent ductus arteriosus. Postoperatively he is extubated but his chest x ray reveals an
elevated left hemi- diaphragm. For this patient, the best treatment is

• A. diaphragmatic plication

• B. intubation

• C. implantation of phrenic nerve stimulator

• D. observation

• E. phrenic nerve grafting


• While the technique of emergency thoracotomy is fairly standard,
the indications for performing surgery remain a source of
controversy. Which one of the following is an absolute indication
for emergency room thoracotomy?
• A. Aortic transection
• B. Blunt thoracic injuries with no witnessed cardiac activity
• C. Chest tube loss of ≤ 500 mls
• D. Penetrating thoracic injury with responsive hypotension
• E. Traumatic arrest secondary to penetrating thoracic injury with
previously witnessed cardiac activity
• Which of the following may Not be amenable to treatment via VATS ? Select Best Answer

A. Retained hemothorax

B. Persistent air leak > 7 days

C. Empyema

D. Active hemorrhage from a pulmonary artery injury

E. Displaced rib fractures

• The optimal resection for a right upper lobe metastatic colon cancer nodule: Select Best Answer

A. Wedge resection with negative margin

B. Segmentectomy

C. Lobectomy

D. This lesion is surgically unresectable


• The beast method of identifying and localizing traumatic tracheobronchial injuries is : Select Best
Answer

A. CXR

B. CT of the chest

C. MRI

D. Bronchoscopy

• Which type of cancer is the most commonly resected after metastasis to the lungs ? Select Best
Answer

A. Breast cancer

B. Melanoma

C. Sarcoma

D. Colorectal cancer

E. Prostate cancer
• What is the best step in management for an 80 year old male with adult-
onset asthma and worsening dyspnea on exertion wheezing ? Select Best
Answer
A. Observation
B. PET scan
C. Radiation Therapy
D. Bronchoscopy
• which of the following lobes has the most variable pulmonary arterial
anatomy ?
A. Right upper lobe
B. Right middle lobe
C. Right lower lobe
D. Left upper lobe
• Accurate staging and restaging of primary tumours and mediastinal nodes in patients
with lung cancer is of significant importance . Which of the following statements
regarding lung cancer staging is correct ?

A. MRI has no advantage over CT scan

B. Less than 20% of patients with a new diagnosis of lung cancer will have metastatic
disease

C. Percutaneous transthoracic needle biopsy has a high false positive rate

D. Transbronchial needle aspiration is particularly effective for paraoesophageal node


biopsy

E. VATS sensitivity and specificity approaches 100%


• Hypercalcemia is most commonly associated with : Select Best Answer
A. Small cell lung cancer
B. Adenocarcinoma
C. Squamous cell carcinoma
D. Carcinoid tumors
• A 24 year old female presents with recurrent right-sided pneumothorax . The
pain , although less intense, occurs during her menstrual cycle . This is
associated with : Select Best Answer
A. Myositis
B. Lupus
C. Endometriosis
D. honor's syndrome
E. Polycystic Ovarian Syndrome
• The most common complication of surgical ligation of PDA is :
• Select Best Answer
A. Phrenic nerve injury
B. Recannalation of the ductus arteriosus
C. Chylothorax
D. Left recurrent laryngeal nerve injury
E. Cerebrovascular accident
• Symptoms of low cardiac output does NOT include : Select Best
Answer
A. Cool extremities
B. Pulmonary congestion

C. Oliguria

D. Anemia
E. Altered mental status
• Lesions greater than what size , in diameter , are considered masses
rather than nodules in the lung ?
• Select Best Answer
A. 20 mm
B. 25 mm
C. 30 mm
D. 35 mm
E. 40 mm
• Which of the following radiographic characteristics makes a pulmonary nodule less likely to
be malignant ?
• Select Best Answer
A. Irregular boundary
B. Lobulated boundary
C. Burr- like boundary
D. Smooth margin
E. Spiculated boundary
• A 28 year old man falls and sustains a simple rib fracture . On examination, there is a small
pneumothorax . What is the most appropriate course of action ?
A. Discharge with advice to return if symptoms worsen
B. Insertion of chest drain
C. Admission for observation
D. CT scanning of the chest
E. Thoracocentesis
• You see a patient damage to the left cervical sympathetic chain
ganglia as a result of a neck tumour . Which of the following
physical signs would be expected ?
A. Ptosis (hanging of the upper eye lid ) on the left
B. Pupil dilation of the left eye

C. Pale skin on the left side of the face

D. Increased sweat secretion on the left side of the face


E. Lateral deviation of the left eye
• Which of the following is NOT considered an indication for
repair of a pectus excavatum deformity ?
• Select Best Answer
A. A Haller index of 3.5 with poor exercise tolerance

B. A Haller index of 2.5 with decreased FEV1


C. Severe pectus excavatum with mitral valve prolapse

D. Severe pectus excavatum with atypical chest pain.



 Which of the followings are true
regarding the thoracic duct?
A. It commences at the level of the second lumbar vertebra
B. In the posterior mediastinum it lies between the
descending thoracic aorta and the azygous vein
C. Lymph transport in the thoracic duct is mainly due to the
action of breathing
D. It is aberrant in almost 15% of patients
E. Patients who have chylothorax after lung resection rarely
require operative repair of the duct
TTTFT aberrant in almost 4% of patients,
Which of the following is an
Indications of chest tube insertion?
• Huge lung cyst with mediastinal shift
• Complete consolidation collapse of the lung
• Huge lung abscess with air fluid level
• Pneumomediastinum
• Pneumothorax
When to remove Chest Tube?
@_After 5 days
@_when became not functioning
@_After complete lung inflation with
No air leak > 24hrs and drainage < 100
cc
@_When discharge became < 100ml/
day
The most common complication
of surgical ligation of PDA is :
• Select Best Answer
A. Recannalation of the ductus arteriosus
B. Chylothorax
C. Left recurrent laryngeal nerve injury
D.Phrenic nerve injury
E.Cerebrovascular accident
 Which of the following is false about
Haller index HI?
@- HI=(A/B) A=Transverse diameter at the level
where deformity is deepest, B= Anterior-posterior
diameter at the same level.
@-HI used to measure the degree of deformity of
PC.
@-The HI is 2.6±0.4 for healthy people.
@-when it is greater than 3.2, it is considered as
severe deformity and indication of surgery.
 All of the following are true about chest
wall tumor except
 Presented as asymptomatic or Painful swelling of the chest
wall
 C/O from general manifestation with S,S. of the primary lesion.
 Metastatic tumor of the rib is the most common malignant
chest wall tumor
 Tissue diagnosis will allow for appropriate staging of the
primary tumor and subsequent management
 All chest wall tumors are treated with wide resection with or
without reconstruction.
A case of postpneumonectomy Rt side empyema with BPF
coughing of huge amount of purulent sputum related to
posture, how you can manage?
Recurrent BPF after surgical treatment of Pt with SSP with
prolonged air leakage, how you can manage?
Q22 Which of the following statements regarding the embryology of the heart and great vessels are
correct?
A.The ductus arteriosus is derived from the fourth branchial arch artery
B.The distal portion of the aortic arch is derived from the sixth branchial arch artery
C.The right recurrent laryngeal nerve lies in relation to the distal portion of the right sixth branchial
arch artery
D.The bracheocephalic artery is derived from the right horn of the aortic sac
E.The left subclavian artery is formed from the left seventh cervical intersegmental artery

FFFTT
The embryo has two dorsal aortae that communicate with an aortic sac via several pairs of
branchial arch arteries. The right dorsal aorta largely and the first,second and fifth pairs of arch
arteries involute.The third pair of arch arteries forms the common and proximal internal carotid
arteries. The left fourth arch artery becomes part of the aortic arch while the right fourth arch
artery becomes the root of the right subclavian artery. The left subclavian artery is derived from an
intersegmental artery arising from the dorsal aorta. The brachiocephalic trunk arises from the
aortic sac.(Refer to The circulatory system in:Moore KL and Persaud TVN.The developing human:
clinically orientated embryology,Sixth Edn.WB Saunders Company,1998 for more detail.)
Q56 Which of the following regarding the blood supply to pedidied tissue
flaps used in chest wall reconstructions are correct?
CA.The blood supply of the rectus abdominis muscle flap is the superior
epigastric artery
CB.The blood supply of the serratus anterior muscle flap is the lateral
thoracic artery
C.The blood supply of the pectoralis major muscle flap is the
thoracoacromial artery,when used as a turn over flap
D. The blood supply of the latissimus dorsi muscle flap is the thoracodorsal
artery
E.The blood supply of the omental flap is based on eiter the right or left
gastroepiploic arteries
TTFTT
The pectoralis can be used as a turnover flap based on the medial sternal
perforators or rotated into the sternal wound based on the thoracoacromial
system.
Q53 Which of the following are true of FDG-PET scanning?
CA.A four hour fasting period is recommended before an FDG-PET
study
B. F used in FDG-PET scanning has a half life of ≈110 min.
C.FDG is taken up by tumour cells and hydrolysed
D.The spatial resolution of PET is better than that of CT
E. A negative FDG-PET scan is an indication of a benign lesion
TTFFF
Raised serum glucose decreases cellular FDG uptake because both glucose and FDG
compete for the same cell surface receptor.Radionuclides used in PET scanning are
typically isotopes with short half lives. FDG is avidly taken up by tumour cells and
phosphorylated. The spatial resolution of PET is about 7 mm,much lower than that of
other imaging methods, such as CT. A negative FDG-PET scan is not an absolute
indication of a benign leion and carcinoid tumours as well as well-differentiated low
Q50 Differences between the right and left main stem bronchi include
A.The right main stem bronchus is supplied by one bronchial artery
B.The left main stem bronchus is supplied by two bronchial arteries
C.The right bronchial artery arises directly from the aorta
D.The right bronchus is narrower than the left
E.The right main stem bronchus enters its lung lower than the left enters
its lung

TTFFF
The right mainstem bronchus is thus more susceptible to the proximal descending
aorta.
Q19 Which of the following statements are true?
A. Heparin inhibits anti-thrombin IIl
B. Bivalirudin inhibits the catalytic activity of thrombin
C. Ancrod acts by enzymatically digesting thrombin
D. Warfarin inhibits synthesis of clotting factors II/VII/IX/X
E. Low molecular weight heparin inhibits factor Xa
FTFTT
Heparin enhances anti-thrombin IIl. Ancrod acts by enzymatically digesting
fibrinogen.
Q6 Which of the following are true regarding Heparin?
A. It is a carbohydrate that consists of a variably sulphated repeating disaccharide
B. It is a naturally occurring anticoagulant produced by basophils and mast cells
C. It works by inactivating anti-thrombin III
D. It plays an important role in the breakdown of clot
E. It results in the formation of soft clot
TTFFT
Heparin is a member of the glycosaminoglycan family of carbohydrates. It acts as
an anticoagulant, preventing the formation of clot and the extension of existing
clot. Heparin binds to the enzyme inhibitor antithrombin III resulting in its active
site being exposed. The activated form then inactivates thrombin and other
proteases involved in blood clotting, such as Factor Xa. Although it is not a
thrombolytic, heparin prevents the formation of a stable fibrin clot by inhibiting the
activation of the 'fibrin stabilising factor'by thrombin.
Q10 Which statements are true regarding the activated clotting time?
A. It is used when heparin levels are too high to allow monitoring with an APTT
or when a rapid result is necessary to monitor treatment
B. The ACT has good sensitivity and range with high doses of heparin,but its
sensitivity is significantly diminished at lower levels
C. The ACT is measured in international units
D. The ACT may be influenced by a patient's platelet count and platelet function
E. The temperature of the blood my also affect ACT results
TTFTT
The APTT test involves an in-vitro clotting reaction and at high levels of heparin
it will not clot. It is measred in seconds.
Q17 The following are true regarding warfarin
A.It enhances vitamine K epoxide reductase
B.It enhances the activity of protein C and protein S
C. It decreases the synthesis of vitamin K dependent cotting factors II,VII,IX and
X
D.African americans are relatively resistant to warfarin, while asian americans
are more sensitive to its effects
E.Skin necrosis occurs more frequently in women and in patients with pre-
existing protein C deficiency
FFFTT
Warfarin inhibits epoxide reductase diminishing available vitamin K and Vitamin
K hydroquinone, which inhibits the carboxylation of the coagulation factors at
glutamic acid residues, making them incapable of binding to the endothelial
surface of blood vessels.The coagulation factors are produced, but have
decreased functionality due to undercarboxylation. Warfarin activity is
determined partially by genetic factors. Skin necrosis is less common in men
Q74 Which of the following is correct regarding reactions to protamine?
A.A type I reaction is hypotension
B. A type IIA reaction is an anaphylactic reaction
C.A type IIB reaction is an anphylactoid reaction
D.A type III reaction is catastrophic pulmonary vasoconstriction
E. Administration of protamine directly into the left side of the heart
reduces the cardiovascular effects of protamine administration
TTTTT
Q70 Which of the following are true regarding the coagulation cascades?
A. The intrinsic pathway is much less significant to hemostasis
B. The intrinsic pathway is activated by cardiopulmonary bypass
D. Activated factor X, Is the site at which the intrinsic and extrinsic coagulation
cascades converge
E. Thrombin inhibits the coagulation cascade
TTFTT

The intrinsic cascade is initiated when contact is made between blood and
exposed negatively charged surfaces such as occurs on exposure to
biomaterials in cardiac surgery, hyperlipidaemia,and bacterial infections.The
extrinsic pathway is initiated at the site of injury in response to the release
oftissue factor (factor III).The activation of factor VIII to factor VIlla occurs in
the presence of minute quantities of thrombin.As the concentration of thrombin
increases, factor VIlla is ultimately cleaved by thrombin and inactivated. This
dual action of thrombin,upon factor VIII,acts to limit the extent of tenase
complex formation and thus the extent of the intrinsic coagulation cascade.
Q49 Which are true of the papillary muscles and their chordae?
A. The anterior papillary muscle of the right ventricle inserts into anterior and
posterior tricuspid valve leaflets
B. The anterior papillary muscle of the left ventricle is larger than the posterior
one
C. The mitral valve has basal type chordae only associated with the anterior
leaflet
D. Papillary muscle rupture of the mitral valve involves the anterior papillary
muscle more often than the posterior
E. Papillary muscle rupture occurs with a relatively small infarction in half the
cases
TTFFT
Mitral valve basal chordae are only associated with the posterior leaflet,The
posterior papillary muscle is involved in about 75% of cases and the anterior in
Q44 The aortic opening in the diaphragm transmits the following
A.The thoracic duct
B.The vagus nerves
C.The phrenic nerves
D.The inferior vena cava
E.The superior epigastric arteries
TFFFF
It is located approximately at the level of the twelfth thoracic vertebra and
transmits the aorta and thoracic duct and often the the azygous vein.
Q45 Which of the following are correct regarding the trachea?
A.The adult trachea is 20 cm from the cricoid cartilage to the carina
CB.It consisits of 18-22 cartilaginous rings
C.Its mucosa is of a ciliated-columnar type
D.The blood supply to the trachea travels in lateral tissue pedicles
E.The muscular tissue consists of two layers of striated muscle

FTTTF
The adult trachea is 11 cm long at approximately two rings/cm of length. During
tracheal mobilisation preserving the lateral blood supply and a tension free
anastomosis is paramount to successful outcome.The muscular tissue consists
of two layers of non-striated muscle,an outer longitudinal and inner transverse
(Trachealis muscle) that forms a thin layer which extends transversely between
the ends of the cartilages.
Q40 Which of the following statements regarding functional
differences between venous conduits and arterial conduits are correct?
A. Veins have a poorly developed internal elastic lamina
B. Heparan sulphate is more expressed in veins than in arteries
C. Production of nitric oxide is lower in veins than in arteries
D. Production of prostacyclin is highe in veins than in arteries
E. Thrombin vasoconstricts saphenous vein whilst dilating internal
mammary arteries
TFTFT
Q33 Which of the following are true regarding the phrenic nerves?
A.They arise from the dorsal segments of the 3rd to 5th cervical dorsal rami
B.They cross the second part of the subclavian arteries bilaterally
C.They enter the thorax between the subclavian artery and vein
D.The left phrenic nerve pierces the central tendon to supply the left
hemidaiphragm
cE.The accessory phrenic nerve joins the main nerve at the root of the neck
FFTFT
The left phrenic nerve crosses the first part of the subclavian artery.The left phrenic
nerve piercesthe dome of the left hemidiaphragm anterior to the centra tendon.
Q16 Which of the following statements regarding the anatomy of the tricuspid
valve are correct?
A.The normal tricuspid valve usually has three leaflets and three papillary
muscles
B.Congenital apical displacement of the tricuspid valve is called Ebstein's
anomaly and typically causes significant tricuspid regurgitation
C.The orifice is smaller than the mitral orifice and is circular
D.The septal leaflet is the largest leaflet
E. The membranous septum and contained A-V conduction system lies
beneath the septal leaflet
TTFFT
Ebstein's anomaly is a congenital malformation of the heart characterised by
apical displacement of the septal and posterior tricuspid valve leaflets,leading to
atrialisation of the right ventricle with a variable degree of malformation and
displacement of he
Q14 Regarding the central venous pressure
A. It measures the right atrial pressure
B. It relates directly to the right ventricular afterload
C. In a ventilated patient on PEEP the true CVP is obtained by subtracting the
PEEP from the CVP reading
D. During the 'v'wave the tricuspid valveis open
E. Large 'a'waves may occur in tricuspid regurgitation
TFFFF
Subtracting the PEEP from the CVP reading tends to underestimate the actual
value.On the other hand excessive PEEP can severely reduce the CVP. Large 'a'
waves occur in tricuspid stenosis,pulmonary stenosis and pulmonary
hypertension. Cannon 'a'waves occur when the right atrium contracts against a
closed tricuspid valve as occurs in complete heart block.
The images are of a 17-year-old woman with Marfan syndrome. She had
moderate preoperative aortic valve regurgitation.
The optimal surgical procedure for this patient is
• A aortic root replacement with a pulmonary autograft
• B aortic root replacement with a valved composite graft
• C aortic valvuloplasty and ascending aorta replacement
• D valve-sparing aortic root replacement

. A conservative and safe approach is the Bentall procedure, which involves


replacing the aortic root and valve with a valved composite graft and coronary artery
button reimplantation
The coronary artery anatomy most likely to be associated with sudden
death in a previously asymptomatic adolescent is
• A left anterior descending coronary artery branching from the right
coronary artery
• B origin of the left coronary artery from right aortic sinus of Valsalva
• C origin of the right coronary artery from left aortic sinus of Valsalva
• D right coronary artery-to-right ventricle fistula
Coronary artery anomalies are the second leading cause of sudden death in older children and young
adults. In particular, anomalous aortic origin of a coronary artery (AAOCA) from the opposite sinus of
Valsalva with an intramural course is the primary coronary cause of sudden death in previously
asymptomatic adolescents. In pathology series the predominant anatomy in sudden death patients is
origin of the left coronary artery from the right sinus (see pictures). This configuration is considerably
more rare than origin of the right coronary from the left sinus. Most practitioners agree that surgical
intervention is indicated if a patient presents with signs and/or symptoms that suggest myocardial
ischemia. It is more common to recommend operative repair if this is an anomalous left coronary
artery than an anomalous right coronary artery. There is some evidence that a longer intramural
coronary length correlates with symptoms. The most common surgical intervention for AAOCA from
the adjacent sinus of Valsalva is coronary artery unroofing (see illustration). The Congenital Heart
Surgeons’ Society has developed a prospective registry to evaluate the outcome of surgical
intervention in these patients.

Origin of the left anterior descending coronary artery from the right coronary artery (single aortic orifice)
is present in 5%-8% of patients with tetralogy of Fallot (TOF). This coronary pattern is usually not
obstructed and has not been associated with sudden death. However, intraoperative injury during
repair of TOF can be lethal. A coronary artery-to-right ventricle fistula causes a left-to-right shunt, but
this has not been associated with myocardial ischemia.
A 20-year-old otherwise healthy man presented with chest pain associated with
vigorous exercise. He recently had an episode of syncope while playing
basketball. Initial transthoracic echocardiography demonstrated anomalous
aortic origin of the right coronary artery from the left sinus of Valsalva. A
subsequent CT angiogram confirmed the presence of a right coronary artery
arising from the left sinus, with an intramural course through the wall of the
aorta, traveling between the aorta and the pulmonary artery.

Which of the following is the best option for treatment?

• A coronary artery bypass grafting


• B coronary reimplantation
• C percutaneous stenting
• D unroofing the artery
The diagnostic imaging ''gold standard '' for aortic
injury is :
• Select Best Answer
A.Computed Tomography
B.Magnetic Resonance Imaging
C.Aortography
D.Endovascular ultrasound
E.CXR
What are the indications for an intra-aortic balloon pump in the peri-
operative period with regards to cardiac surgery?
• Elective in patients with significantly impaired pre-operative cardiac function.
• Peri-operative ischaemia.
• Unloading of the left ventricle following a mechanical complication of
myocardial infarction, such as a ventricular septal rupture or acute myocardial
regurgitation.
• Low cardiac output syndrome, unresponsive to inotropic support.
• Acute myocardial deterioration, as a bridge to revascularization, left
ventricular assist device or transplantation.

• Contra-indications to intra-aortic balloon pump insertion include the


presence of:
a) aortic regurgitation;
b) aortic dissection;
c) severe peripheral atherosclerosis.
Which of the following statements regarding functional
differences between Venous conduits and Arterial conduits are
correct?
A. Veins have a poorly developed internal elastic lamina
B. Heparan sulphate is more expressed in veins than in arteries
C. Production of nitric oxide is lower in veins than in arteries
D. Production of prostacyclin is higher in veins than in arteries
E. Thrombin vasoconstricts saphenous vein whilst dilating internal
mammary arteries

TFTFT
The coronary artery anatomy most likely to be associated with sudden
death in a previously asymptomatic adolescent is
• A- left anterior descending coronary artery branching from the right
coronary artery
• B- origin of the left coronary artery from right aortic sinus of Valsalva
• C- origin of the right coronary artery from left aortic sinus of Valsalva
• D- right coronary artery-to-right ventricle fistula
What are the advantages of repairing the mitral valve
rather than replacing it?
• As compared with mitral valve replacement, mitral valve repair is
associated with greater freedom from mortality (operative and long-
term), structural valve deterioration, re-operation, infective
endocarditis, thrombo-embolism and haemorrhage.

• As more of the sub-valvular apparatus is preserved, mitral valve repair


is better at maintaining left ventricular geometry and hence is
associated with less left ventricular dysfunction compared with mitral
valve replacement.
Which of the following statements regarding the anatomy of the
tricuspid valve are correct?
A. The normal tricuspid valve usually has three leaflets and three
papillary muscles
B. Congenital apical displacement of the tricuspid valve is called
Ebstein's anomaly and typically causes significant tricuspid
regurgitation
C. The orifice is smaller than the mitral orifice and is circular
D. The septal leaflet is the largest leaflet
E. The membranous septum and contained A-V conduction system
lies beneath the septal leaflet

TTFFT Ebstein's anomaly is a congenital malformation of the heart characterised by apical displacement of the septal and posterior tricuspid valve leaflets,leading to
atrialisation of the right ventricle with a variable degree of malformation and displacement of he
What factors should be considered when choosing between a
mechanical and bioprosthetic aortic valve (Figures )?
• Prosthesis type does not influence survival, thrombo-embolism or infective endocarditis rates.
• Bleeding is more frequent with mechanical valves.
• SVD is more common with bioprosthetic valves.
• Mechanical valves are the preferred choice if:
a) the patient is already on warfarin with another mechanical valve;
b) the patient is young (age <60) and does not want another operation.
• Bioprosthetic valves are the preferred choice if:
a) the patient is aged >65
b) the patient wants to avoid warfarin (patient preference, contraindication to warfarin, woman
of child-bearing age, young patient with active lifestyle, recent GI bleed, awaiting future
operation for malignancy, etc).

What valve should be used in patients with renal failure on hemodialysis?


• The AHA guidelines in 1998 suggested using a mechanical prosthesis; however, the AHA guidelines
in 2006 made no recommendation.
• It has been suggested that most modern bioprosthetic valves will last longer than these patients
with end-stage renal failure, hence by implanting a bioprosthetic valve, it is possible to avoid the use
of long-term anticoagulation in these difficult cases..
What are the indications for an emergency resuscitative thoracotomy?
• According to the Advanced Trauma Life Support (ATLS®) guidelines,
following penetrating trauma (not blunt trauma) within 10 minutes of
pulseless electrical activity (PEA).
• It is performed by a left anterolateral thoracotomy through the 4th
intercostal space.
• Through this access, the therapeutic options include:
a) evacuation of the pericardium;
b) open cardiac massage;
c) clamping of the descending aorta (to stop distal bleeding and increase
coronary and cerebral perfusion).
What are the standard anticoagulation regimes following
cardiac surgery?
• According to the American Heart Association (AHA) guidelines, the
following INRs (international normalized ratios) are recommended
using warfarin:
a) coronary endarterectomy - 2.0-3.0;
b) atrial fibrillation - 2.0-3.0;
c) mechanical aortic valve - 2.0-3.0;
d) mechanical mitral valve - 2.5-3.5.

• For patients with bioprosthetic valves, aspirin alone can be used or 3


months of warfarin (with optimal INR 2.0-3.0) followed by aspirin alone.
Which statements Regarding the central venous pressure are
true?
A. It measures the right atrial pressure
B. It relates directly to the right ventricular afterload
C. In a ventilated patient on PEEP the true CVP is obtained by
subtracting the PEEP from the CVP reading
D. During the 'v’ wave the tricuspid valve is open
E. Large 'a’ waves may occur in tricuspid regurgitation

TFFFF

Subtracting the PEEP from the CVP reading tends to underestimate the actual value.On the other hand excessive PEEP can severely reduce the CVP. Large 'a' waves
occur in tricuspid stenosis,pulmonary stenosis and pulmonary hypertension. Cannon 'a'waves occur when the right atrium contracts against a closed tricuspid valve as
A young female with a long term intravenous jugular
catheter is waiting for surgery. Two days later she
presents with a cold left leg.
The best test for this patient now is:
a. Femoral angiogram
b. Cardiac catheterization
c. Echocardiogram
d. Venogram

Please read this question well. The patient has an intravenous line and she most likely has a clot in it. For the clot to cause a cold leg, it means that patient must have a PFO or ASD. Do not rule out PFO. This patient needs an ECHO to diagnose paradoxical
embolus via a PFO.
Which of the following are true regarding the coagulation
cascades?
A. The intrinsic pathway is much less significant to hemostasis
B. The intrinsic pathway is activated by cardiopulmonary bypass
D. Activated factor X, Is the site at which the intrinsic and extrinsic
coagulation cascades converge
E. Thrombin inhibits the coagulation cascade

TTFTT

The intrinsic cascade is initiated when contact is made between blood and exposed negatively charged surfaces such as occurs on exposure to biomaterials in
cardiac surgery, hyperlipidaemia,and bacterial infections.The extrinsic pathway is initiated at the site of injury in response to the release oftissue factor (factor III).The
activation of factor VIII to factor VIlla occurs in the presence of minute quantities of thrombin.As the concentration of thrombin increases, factor VIlla is ultimately
cleaved by thrombin and inactivated. This dual action of thrombin,upon factor VIII,acts to limit the extent of tenase complex formation and thus the extent of the
Q6 Which of the following are true regarding Heparin?
A. It is a carbohydrate that consists of a variably sulphated repeating
disaccharide
B. It is a naturally occurring anticoagulant produced by basophils and
mast cells
C. It works by inactivating anti-thrombin III
D. It plays an important role in the breakdown of clot
E. It results in the formation of soft clot

TTFFT

Heparin is a member of the glycosaminoglycan family of carbohydrates. It acts as an anticoagulant, preventing the formation of clot and the extension of existing
clot. Heparin binds to the enzyme inhibitor antithrombin III resulting in its active site being exposed. The activated form then inactivates thrombin and other
proteases involved in blood clotting, such as Factor Xa. Although it is not a thrombolytic, heparin prevents the formation of a stable fibrin clot by inhibiting the
activation of the 'fibrin stabilising factor'by thrombin.
Which statements are true regarding the activated clotting
time?
A. It is used when heparin levels are too high to allow monitoring with an APTT
or when a rapid result is necessary to monitor treatment
B. The ACT has good sensitivity and range with high doses of heparin, but its
sensitivity is significantly diminished at lower levels
C. The ACT is measured in international units
D. The ACT may be influenced by a patient's platelet count and platelet function
E. The temperature of the blood my also affect ACT results

TTFTT

The APTT test involves an in-vitro clotting reaction and at high levels of heparin it will not clot. It is measred in seconds.
Which of the following is correct regarding reactions to
protamine?
A. A type I reaction is hypotension
B. A type IIA reaction is an anaphylactic reaction
C. A type IIB reaction is an anphylactoid reaction
D. A type III reaction is catastrophic pulmonary vasoconstriction
E. Administration of protamine directly into the left side of the heart
reduces the cardiovascular effects of protamine administration
Which statements of the following are true regarding warfarin?
A. It enhances vitamine K epoxide reductase
B. It enhances the activity of protein C and protein S
C. It decreases the synthesis of vitamin K dependent cotting factors II,VII,IX and
X
D. African americans are relatively resistant to warfarin, while asian americans
are more sensitive to its effects
E. Skin necrosis occurs more frequently in women and in patients with pre-
existing protein C deficiency

FFFTT

Warfarin inhibits epoxide reductase diminishing available vitamin K and Vitamin K hydroquinone, which inhibits the carboxylation of the coagulation factors at
glutamic acid residues, making them incapable of binding to the endothelial surface of blood vessels.The coagulation factors are produced, but have decreased
functionality due to undercarboxylation. Warfarin activity is determined partially by genetic factors. Skin necrosis is less common in men and in patients with
protein S deficiency.
•Tension pneumothorax :-
• is a life-threatening complication that requires immediate recognition and
urgent treatment.
• Tension pneumothorax is caused by the development of a valve-like leak in the
visceral pleura, such that air escapes from the lung during inspiration but cannot re-
enter the lung during expiration. This process leads to an increasing pressure of air
within the pleural cavity and hemodynamic compromise because of impaired venous
return and decreased cardiac output.
• Treatment is high flow oxygen and emergency needle decompression with a
cannula inserted in the second intercostal space in the midclavicular line.
Intercostal drain is then inserted after decompression.
• Emergency treatment must be based on a
clinical diagnosis before radiological confirmation, because of
life threatening hemodynamic compromise.
• Radiographic features suggesting tension pneumothorax includemediastinal
shift away from the affected side,inversion of the hemidiaphragm andwidening of
intercostal spaces from the increased pressure within the affected hemithorax.
•Tension pneumothorax
• Tension pneumothorax is a medicalemergency that requires
immediate treatment.
•Needle decompression of the chest, also known as needle
thoracostomy, to allow for the release of air from the chest cavity is the
preferred method of treatment.
• Alternatively, achest tube may be inserted to allow for continued
evacuation of air and reduction of pressure on the lungs.
•Treatment of tension pneumothorax is done on an emergency basis
and should be performed before confirmatory radiologic
studies.
• Needle decompression for emergency cases
• chest tube is indicated for stable patients.
Management
1-Conservative management Observation
(unilateral, small <20% pneumothorax, no apical blebs).
2-Trial of needle aspiration if minimal symptoms
and moderate PTx.
3-Tube thoracostomy if compromised or very large PTx.•
4-Surgery
• Figure 1 Light’s index for estimating the size of pneumothorax. (A) Lung diameter and (B)
hemithorax diameter (both measured at the level of pulmonary hilum).
size of the pneumothorax according to Light’s index [size of the pneumothorax (in % ) = [(1 - (L 3/
HT 3)] × 100, whereL andHT are the diameters of the lung and the hemithorax, respectively, both
measured at the level of the pulmonary hilum] (Fig. 1),
• According to Light’s index (Figure 1):
a)if PTX is less than 15% conservative,
b)if PTX 15%-30% NA,
c)if PTX more than 30% ICD
According to British Thoracic Society:
measurement from the lung margin to chest wall at
the level of the hilum.
a)a) Small less than 1 cm conservative,
b)b) Medium 1-2 cm NA,
c) large more than 2 cm ICD
• Current Management Guidelines:
• The two main goals of management in SP aretreatment of a
PAL and/or reducing recurrence risk .
• In the first episodes of PSP, simple needle aspiration(NA) or
pleural drainage are the current settled first line therapies with an
immediate success rate of 59.3% and 68% respectively.
• Surgery is generally reserved for episodes of pneumothorax that
do not resolve, recurrences, pneumothorax associated with
hemothorax, bilateral pneumothorax or for occupations at risk.
• Two surgical approaches are possible : open thoracotomy (OT)
and video-assisted thoracoscopic surgery (VATS).The surgical
technique then includes the resection of parenchymal bullae and
pleurodesis(chemical or mechanical)
• The BTS guidance outlines the initial management of an SSP:
in the absence of hemodynamic compromise, with three options:
• 1. Conservative inpatient managementif the SSP is <1 cm at the hilum
• 2. Needle aspiration (NA) and admission for observationif the SSP is 1–2
cm at the hilum
• 3. Chest tube drainage (CTD) if the patient is unstable, breathless,if SSP
is >2 cm at the hilum, or >1 cm at the hilum after an attempted aspiration .
If hemodynamic compromise is present :
• Urgent needle decompression with a 14-gauge cannula in the 2nd
intercostal space mid-clavicular line or 4th/5th intercostal space mid-
axillary line followed by CTD insertion is recommended. Oxygen
supplementation may help accelerate the resolution of a pneumothorax,
however care must be taken in the context of a SSP as many patients will
be at risk of type 2 respiratory failure.
• Open procedures
were performed bythoracotomy in the fourth or fifth inter costal
space.Pleurodesis procedures were either mechanical or chemical.
Mechanical pleurodesis consisted of pleural abrasion with or without
apical pleurectomy. Chemical pleurodesis consisted of applications
of tetracycline, bleomycin or talc powder.
Abullectomy was performed when blebs were found. At the end of
the procedure, one or twochest tubes were placed in the pleural
cavity for postoperative drainage.
Patients left the hospital 1 day after removal of the chest tube, and
whenno residual pneumothorax was seen on the follow-up chest X-
ray.
Surgery is indicated in the following cases;
First episode: prolonged air leak (>72h) , hemothorax, bilateral pneumothoraces, residual
collapse of lung, occupational hazard, PTx secondary to giant bulla, previous contralateral
pneumonectomy.
Recurrence of PTx: the aim is to resect the blebs or bullae and obliterate the pleural space with
adhesions either using chemical or abrasion pleurodesis or parietal pleurectomy (apical or full).

It may be performed through a minithoracotomy, axillary incision, or thoracoscopically.


Recurrence rate
<2%following surgical pleurectomy via minithoracotomy.
5% after VATs pleurodesis,
5–10% following chemical pleurodesis.
Which of the following statements are true?
A. Tension pneumothorax can cause haemodynamic
compromise.
B. Pleural effusions due to cardiac failure, renal failure, hepatic
disease, inflammatory disease and malignancy have different
protein content.
C. Infection of the pleural space (empyema) results from
iatrogenic and non-iatrogenic causes.
D. Video-assisted thoracoscopic surgery (VATS) plays a major
role in the management of pleural space diseases.
E. Chest drains are no longer critical to the management of chest
disease
•A= Thoracotomy
•B= Chest drain insertion
•C= Bronchoscopy
•D= CT scan of the chest
•E= None of the above
• 1- A middle-aged male Pt presents with spontaneous recurrent
haemoptysis, and clinical Ex. reveals no abnormal findings. His
CXR is normal. What is the procedure of choice?
• 2- After reading the information booklet about her proposed
procedure, an anxious 45-year-old female patient being
consented for a procedure is worried about the post-procedural
complications of pain and possible rib fractures. What
procedure is this patient planned to have?
• 3- Following a right pneumonectomy, the postoperative recovery
of a 52-year-old male patient is complicated by BPF, which
presents with pyrexia, expectoration of large amounts of
purulent sputum and a high fluid level on CXR. In addition to
positioning the patient to lie on the operated side, what
procedure is urgently required?
• 4- A mediastinal mass is found incidentally on the
CXR of a 65-year-old female non-smoker who is
otherwise healthy, with no significant past medical
history. Which option would you recommend to
this patient at this time?
• 5- Which procedure is required to assess the
fitness for lung resection of a 73-year-old male
with left upper lobe lung cancer and emphysema?
• 1C Haemoptysis is commonly caused by bronchopulmonary trauma, infection or neoplastic (benign and
malignant) lesions of pulmonary system. The underlying pathologies are usually in direct or indirect
communication with the bronchial tree. Bronchoscopy affords the ability to visualise the lesion, the potential
to obtain biopsies or treat, or the insight to plan further treatment. The flexible bronchoscope can be
advanced into segmental bronchi and is useful for obtaining sputum and tissue biopsies. As the calibre is
small and suction is limited, flexible bronchoscopes may not have optimal diagnostic and therapeutic yield
soon after an episode of haemoptysis because blood clots obscure visualisation. Rigid bronchoscopy
overcomes these setbacks but requires general anaesthesia.
• 2A Thoracotomy involves muscle cutting, rib spreading and parietal pleura breeching. Sometimes rib
fractures occur and the intercostal nerves are bruised during rib spreading. In the early postoperative period,
therefore, thoracotomy pain can be severe and difficult to control. The functional consequence of post-
thoracotomy pain leads to other complications such as impairment of mobilisation, normal breathing and
gas exchange. The strategies commonly used to manage early post-thoracotomy pain include oral
analgesia, and either intravenous opiates used as patient-controlled analgesia (PCA) or local anaesthesia
delivered via catheters into the paravertebral or extrapleural space.
• 3B Morphological changes such as mediastinal shift, elevation of the hemidiaphragm and crowding of the
ribs occur after pneumonectomy to contract the pneumonectomy space, which fills with tissue fluid.
Dehiscence of the bronchial stump occurs in bronchopleural fistula to establish communication between
colonised bronchial tree and the sterile pneumonectomy space. Invariably, the pneumonectomy space and
fluid get infected in bronchopleural fistula. Signs of systemic infection (pyrexia) with clinical (expectorating
purulent sputum) and radiological (high fluid level) evidence of infected collection in the chest warrant
immediate chest drain insertion to control the source of sepsis. Further management of bronchopleural
fistula is undertaken in specialised centres.
• 4D A mediastinal mass on chest radiograph deserves further investigation with CT of the
chest to define the site, size, nature and attachments of the mass and surrounding structures,
and, where possible, to obtain a guided percutaneous biopsy for histological diagnosis. On the
basis of the location in the mediastinum, the possible cause of the mass can be suspected.
Common mediastinal masses in the different parts of the mediastinum are:
• superior mediastinum – lymphoma, thyroid and parathyroid
• anterior mediastinum – thymoma, lymphoma and germ cell tumour
• middle mediastinum – cystic lesions, lymphoma and mesenchymal tumours
• posterior mediastinum – neurogenic tumours, cystic lesions and mesenchymal tumours.
• 5E In order to determine the fitness of patients for lung resection, and the extent of resection
they can tolerate, pulmonary function test is necessary. Peak expiratory flow rate, forced
expiratory volume in 1 sec (FEV1), forced vital capacity (FVC) and arterial blood gases provide
useful information. In general, a patient is fit for lobectomy if the FEV1 is >1.5 L and
pneumonectomy if >2.0 L.
Stages
Stage I: acute exudative phase Typically occurs 2–5 days after a pneumonia. Accumulation of
fluid with low cellular content and viscosity.

Characterized by low WBC, LDH, glucose, and a normal pH.


Can be successfully treated with antibiotics only.

Stage II: fibrinopurulent phase Typically occurs 5–14 days after a pneumonia. Turbid or purulent
fluid with heavy fibrin deposits leading to loculations and septations. May have bacterial invasions,
and high numbers of polymorphonuclear leukocytes (PMNs) and lymphocytes.

Characterized by low pH, glucose, and increased LDH. Antibiotics and chest tube drainage is
required, or VATS decortication.

Stage III: chronic organizing phase Lung trapping by collagen, visceral and parietal pleural peel
with ingrowth of fibroblast and capillaries. Antibiotics and aggressive decortications, generally by
thoracotomy.
Management
The aim of treatment is to drain and sterilize the space, and re-
expand the lung
Stage I
Sensitivity-appropriate antibiotics.
Stage II
Antibiotics and Chest tube drainage (small or large bore).
Thrombolytics most likely to be effective during early stage empyema, and should
be reserved for patients who are poor surgical candidates.
Stage III and failure of stage II to resolve
If pleural space drainage is ineffective, or the effusion has loculated appearance on
imaging, aVATS decortication should be performed without delay If complete lung
expansion is not achieved by VATS, then should convert to
open thoracotomy :
Open decortication indicated for late stage II, stage III, and incomplete lung
expansion with VATS.
Chest wall window (Eloesser flap ) may be required if lung cannot expand and
empyema chronically reaccumulates, or in the case of a BPF that cannot be
closed.
Diagnosis of BPF
General manifestations: fever, toxemia
Local manifestations: Patient may start expectorating foul watery
fluid most frequently after lying with operated side uppermost.
Sometimes patients complain of new ‘gurgling’ sensation in chest.

New or falling air-fluid level in pneumonectomized pleural space on


CXR and CT.
Bronchoscopy confirms diagnosis: allows the stump and any
defects in it to be visualized directly, as well as permitting bronchial
toilet.
Management of BPF
Drainage of pleural space using tube thoracostomy, antibiotic therapy, and
eventual obliteration of space.
Bronchoscopy can confirm the presence or absence of bronchopleural fistula.
A surgical window may be required in order to provide adequate drainage for
obliteration of the space.
Closure of window is possible following complete sterilization of the thoracic
cavity.
Other options include thoracoplasty or obliteration of the empyema space.
Management
first involves preventing the patient from aspiration of pleural fluid through the BPF, and treating
the infection Nurse the patient with the operated side lowest to prevent spillage into the airway.
Immediate chest tube after discovery of new air fluid level in the pneumonectomized pleural
space.
Occasionally this is all that is required to drain the space and allow the defect to close.
Failure to close and contamination of the pleural space requires:
Creation of achest wall window and primary closure of the BPF .
Serial debridement of the infected space with either continuous packing or wound vacuum
dressing.
Filling of space either with space-occupying tissue (omentum, muscle) or antibiotic solution
and primary closure of chest wall (Clagett procedure)
Early fistulas are usually associated with technical difficulty at surgery, repeat thoracotomy and
resuturing of stump is indicated.
Fistulae occurring >2 weeks postoperatively are usually associated with empyema: iv antibiotics
should be administered.
 Which of the followings are true
regarding the thoracic duct?
A. It commences at the level of the second lumbar vertebra
B. In the posterior mediastinum it lies between the
descending thoracic aorta and the azygous vein
C. Lymph transport in the thoracic duct is mainly due to the
action of breathing
D. It is aberrant in almost 15% of patients
E. Patients who have chylothorax after lung resection rarely
require operative repair of the duct
TTTFT aberrant in almost 4% of patients,
Male patient 40 years old, smoker, uncontrolled DM with
recurrent diabetic coma, fever, coughing of huge amount of
purulent sputum when laying forward or laying on his left
side, shortness of breath, easy fatigability, anorexia, chest
pain on right side, swelling and tenderness at the right lateral
chest wall, severely pallor (toxic), lower limb edema,
cachexia, decreased air entry on the right side and clubbing
of fingers
What is yourdiagnosis andmanagement ?
Treatment
• I- Medical treatment
• II- Adequate drainage
• III- Surgical treatment
I- Medical treatment
• Symptomatic treatment
• Treatment of the underlying cause
• Use of the appropriate antibiotics
to control infection
• Surgical treatment options
• @resection of the involved segment ,
(segmentectomy or lobectomy)
• @ in case of rupture into the Pl space;
simpleunroofing of the cavity
(Cavernoplasty)
Treatment
• I- Medical treatment
• II- Surgical treatment
• III- Minimally invasive
treatment
III- Minimally invasive treatment
@ Resection via thoracoscopic surgery(VATS)
@Bronchial artery embolization in the case of
severe hemoptysis
“A 48 yo mechanic presents to the ED with pain in his right hand and numbness that
began acutely this morning.
• You notice that he has an absent pulse and petechiae in the first and second digits.
His hand appears dusky and motor function is slightly diminished.
• He notes a similar episode that resolved about a month ago. He does not have a
history of arrhythmias (i.e., AF), diabetes, nor peripheral vascular disease. He does
complain of symptoms consistent with arm claudication with repetitive maneuvers.
• How do you proceed?”
• This scenario gives an example of aTOS – poststenotic aneurysm with distal embolization.
Occasionally acute thrombosis or retrograde thrombosis and cerebral embolization develops.
• Workup includes vascular lab imaging: CT angiography and/or angiography.
• In acute arterial thrombosis, endovascular restoration with regional thrombolysis or
thrombectomy may be possible and should be attempted first. The patient should be
anticoagulated after flow has been reestablished.
• The primary issue (compression of the subclavian artery) can then be addressed more electively.
In elective situations, resection of the aneurysm and bypass or interposition graft is used for
reconstruction (proximal and distal control, heparinization prior to clamping) following first rib
decompression.
“A 28 yo pitcher presents with acute right arm swelling. He was
diagnosed with a subclavianaxillary vein clot on duplex in the ED.
• How would you manage this patient?”
• This is an example ofPaget-Schroetter Syndrome , This syndrome typically affects young athletes
with excessive/repetitive use of the arm (i.e., pitchers, basketball players, weight lifters).
• Compression of the subclavian/axillary vein is caused by compression against the first rib by a
congenitally, laterally displaced costoclavicular ligament along with a hypertrophied ASM.
• Diagnose with venous duplex and venogram.
• Patients should also be worked up for a hypercoagulable state.
• Initial treatment is with catheter directed thrombolysis (access through basilic vein, cross thrombus
with wire and pulse-spray catheter with tPA infusion).
• After successful thrombolysis, the patient is treated with a heparin bridge and started on Coumadin.
Patients with evidence of venous compression should undergo TOS decompression after 3 months
of coumadin therapy.
• Failed thrombolysis is an indication for open thrombectomy and decompression during the same
admission versus long-term coumadin therapy if the patient does not wish to have operative
intervention.
• “Six months after a transaxillary first rib resection, a 55 yo man
with nTOS presents with recurrence of symptoms.
• How would you manage this situation?”
• Recurrent TOS – distinguish true recurrence (initial improvement
followed by recurrence – due to scarring) vs. false (lack of initial
improvement – inadequate surgery – either wrong diagnosis or
wrong procedure (i.e., 2nd rib resection).
• Thorough workup and imaging for assessment of performed
procedure. Start with PT.
• If repeat surgical intervention deemed to be necessary, the
preferred approach is a high posterior thoracoplasty.
“You are seeing a 29 yo woman who complains of right
shoulder and neck pain, occipital headaches, and weakness/
numbness of her right arm over the last several months. She
is seeking disability benefits and requesting an oxycontin
refill (oxycodone is a member of the Opioids (narcotic analgesics)
drug class and is commonly used for Chronic Pain, and Pain).”
“The patient admits to having progressive symptoms over
several months and now has disabling weakness in her right
arm. She is employed as a librarian, but is unable to reach for
books on upper shelves and frequently drops objects from
her hand. She also admits to frequent tension headaches.
• On physical examination she has decreased sensation in her
medial hand and fingers and internal forearm. Hypothenar
atrophy is noted. There is tenderness over the
rightsupraclavicular region.
• EAST and Elvey tests are positive The elevated arm stress
test (EAST) is also performed with the arms elevated to
provoke symptoms.
• The upper extremity tension test (Elvey’s test) checks for
pinching of the nerve at the thoracic outlet. Peripheral nerves
are tested along with range of motion in the arm.
• Chest and C-spine X-rays do not reveal any abnormalities,
MRI is negative. UNCV shows ulnar nerve conduction of 65
m/s and EMG studies were normal.”
• A 50 year old electrician has pain in his upper extremity , and
tingling and numbness in the 4th and 5th digits of his left
hand in his medial forearm. The patient says that most of the
pain and numbness occurs when he is doing electrical work
with his arms overhead . PE shows mild swelling of the left
hand . An X-ray of the neck shows a cervical rib.
• Which of the following is most likely compressed ?
• A- upper trunk of brachial plexus
• B- lower trunk of brachial plexus
• C- subclavian vein
• D- subclavian artery
• Initial management for all patients with a confirmed or suspected diagnosis
of nTOS consists of physical therapy.
• This should focus on stretching the scalene muscles, normalizing posture,
and strengthening muscle of the shoulder girdle.
• NSAIDs, muscle relaxants, and non-narcotic analgesics are adjuncts.
Progress is reassessed in 4-6 wks.
• Two-thirds of the patients with nTOS improve with physical therapy and
avoid surgical intervention.
• Failure of conservative therapy is an indication for surgical decompression.
• Indications for surgical decompression without a trial of physical therapy
includes nTOS with motor deficits (compared to sensory only), mixed
symptoms (both neurologic and vascular symptoms) and vascular
symptoms
(arterial and venous TOS are always immediate surgical indications).
• NTOS is initially managed with a trial of physical rehabilitation for several months.
• – Refractory cases (over 50 %) should be referred for surgical intervention .
• ATOS is more urgent and should be treated surgically.
• – Decompression is usually done via either a transaxillary or supraclavicular
approach.
• – The artery is generally bypassed and any structures that are compressing the
thoracic outlet are resected (the first rib is normally removed and anterior and
middle scalenectomies are performed).
• – If distal embolization has occurred, the patient should undergo either
thrombolysis or embolectomy .
• VTOS is treated with thrombolysis and subsequent surgical decompression.

• • Complications: injury to the phrenic nerve, branches of the brachial plexus, or


thoracic duct, and pneumothorax.
• • Immediate post-operative rehabilitation should follow to maintain range-of-
motion and function of the affected limb
• Indications for surgery include:-
1-failure of conservative measures to attenuate the symptoms caused
by nerve compression after a 3-month period and
2-presence of prolonged conduction velocities in the ulnar or median
nerve.
• Other surgical indications include:-
• (1) the presence of atypical chest pain unrelieved by conservative
management (not related to coronary artery, esophageal, or pulmonary
pathological conditions);
• (2) the presence of hypersympathetic activity;
• (3) the narrowing or occlusion of the subclavian artery with or without
peripheral emboli; and
• (4) thrombosis of the subclavian vein (Paget-von Schrötter syndrome,
effort thrombosis).
• Two main surgical approaches currently employed:
• transaxillary (best for nTOS and vTOS) and
• supraclavicular (best for aTOS).
• posterior transthoracic approach and
• VATS approach have also been described.
TO
S
A young female with a long term intravenous jugular catheter
is waiting for surgery. Two days later she presents with a cold
left leg.
The best test for this patient now is:
a. Femoral angiogram
b. Cardiac catheterization
c. Echocardiogram
d. Venogram
Please read this question well. The patient has an intravenous line and she most likely has a clot in it. For the clot to cause a cold leg, it means that patient must have a PFO or ASD. Do not rule out PFO.
This patient needs an ECHO to diagnose paradoxical embolus via a PFO.
A 34 year old is shot in the chest. ECHO reveals that the bullet is lodged
in the atrial septum. Two days later the patient develops a syncopal
attack and CT scan shows that the bullet is now at the right carotid
bifurcation.
The best approach to this patient is:
a. to remove the bullet from the carotid via a neck incision
b. to open both the chest and neck
c. to do a percutaneous removal of bullet via the femoral artery in the
angio lab
d. to wait and remove the bullet from the carotid at a later time

B)This patient has a paradoxical embolus of a bullet via an atrial septal defect. At this time both the atrial septal defect has to be closed and the bullet must be removed from the neck. The only way the bullet
could have gotten from the right atrium to the right carotid bifurcation is via the atrial septal defect. Both the septal defect and the bullet have to be treated.
What factors should be considered when choosing between a mechanical and
bioprosthetic aortic valve (Figures )?
• Prosthesis type does not influence survival, thrombo-embolism or infective endocarditis rates.
• Bleeding is more frequent with mechanical valves.
• SVD is more common with bioprosthetic valves.
• Mechanical valves are the preferred choice if:
a) the patient is already on warfarin with another mechanical valve;
b) the patient is young (age <60) and does not want another operation.
• Bioprosthetic valves are the preferred choice if:
a) the patient is aged >65
b) the patient wants to avoid warfarin (patient preference, contraindication to warfarin, woman
of child-bearing age, young patient with active lifestyle, recent GI bleed, awaiting future
operation for malignancy, etc).

What valve should be used in patients with renal failure on hemodialysis?


• The AHA guidelines in 1998 suggested using a mechanical prosthesis; however, the AHA guidelines
in 2006 made no recommendation.
• It has been suggested that most modern bioprosthetic valves will last longer than these patients
with end-stage renal failure, hence by implanting a bioprosthetic valve, it is possible to avoid the use
of long-term anticoagulation in these difficult cases..
What are the criteria for extubation following cardiac surgery?
• Cardiac: a) cardiac index >2.2L/min/m2;
b) mean arterial blood pressure >70mmHg;
c) no haemodynamically significant arrhythmias.
• Respiratory: a) partial pressure of oxygen in arterial blood (PaO2) >10kPa and arterial oxygen
saturation (SaO2) >92% with an FiO2 <50%;
b) PEEP <5cm H2O;
c) respiratory rate <20 per minute;
d) pressure support <10cm H2O.
• Mediastinal drainage <50mL/hr with haemoglobin >8g/dL.
• Urine output >1mL/kg/hr.
• Neurological: a) awake with stimulation and no residual neuromuscular blockade;
b) able to lift head off the bed;
c) able to cough and clear secretions to maintain own airway.
• Core temperature >36°C.
• Once the sedation has been stopped, the patient’s ventilator settings are maintained with an FiO2 at
50% and PEEP of 5mmHg, and the IMV rate is reduced by two breaths every 30 minutes checking
arterial blood gases and then removing the endotracheal tube if the patient maintains cardiorespiratory
stability.
Alternatively, a trial of continuous positive airway pressure (CPAP) with the endotracheal tube in situ
is performed to assess whether the patient is able to self-ventilate.
Poor prognostic factors for weaning include:
a) tidal volume <5mL/kg
b) minute volume <10L/min
c) maximum inspiratory pressure <-20cm H2O
d) shallow breathing index (respiratory rate/tidal
volume).
What are the standard anticoagulation regimes following
cardiac surgery?
• According to the American Heart Association (AHA) guidelines, the
following INRs (international normalized ratios) are recommended
using warfarin:
a) coronary endarterectomy - 2.0-3.0;
b) atrial fibrillation - 2.0-3.0;
c) mechanical aortic valve - 2.0-3.0;
d) mechanical mitral valve - 2.5-3.5.

• For patients with bioprosthetic valves, aspirin alone can be used or 3


months of warfarin (with optimal INR 2.0-3.0) followed by aspirin alone.
Which of these statements are true?
• A- Biological valves are obtained from animals (xenograft or heterograft),
dead humans (allograft or homograft) and the patient (autograft).
• B- Mechanical valves are more durable than biological valves.
• C- Lifelong anticoagulation is required for all prosthetic valves.
• D- Age is the only determinant of the choice of prosthetic valve.
• E- Biological valves are not at risk of prosthetic valve endocarditis
Which of the following statements is false?
• A- Cardiopulmonary bypass has brought about remarkable progress in
cardiac surgery.
• B- The cardiopulmonary bypass circuit consists of a venous reservoir,
oxygenator, heat exchanger, filter and roller pump.
• C- Cardiopulmonary bypass is not used outside cardiac surgery.
• D- Patients require full-dose heparin with the use of cardiopulmonary bypass.
• E- Cardiopulmonary bypass can cause serious systemic complications
A 66-year-old male is informed during consent for a procedure for
a cardiac disease that, in addition to a procedure-related mortality
of about 2 per cent, there is a risk of neurological dysfunction of
less than 2 per cent, cardiac arrhythmia up to 30 per cent and
significant bleeding up to 5 per cent.
What condition is this procedure for?
Which of the following are true regarding Heparin?
• A. It is a carbohydrate that consists of a variably sulphated repeating
disaccharide
• B. It is a naturally occurring anticoagulant produced by basophils and
mast cells
• C. It works by inactivating anti-thrombin III
• D. It plays an important role in the breakdown of clot
• E. It results in the formation of soft clot
• TTFFT

• Heparin is a member of the glycosaminoglycan family of carbohydrates. It acts as an anticoagulant, preventing the formation of clot and the
extension of existing clot. Heparin binds to the enzyme inhibitor antithrombin III resulting in its active site being exposed.The activated form then
inactivates thrombin and other proteases involved in blood clotting, such as Factor Xa. Although it is not a thrombolytic, heparin prevents the
formation of a stable fibrin clot by inhibiting the activation of the 'fibrin stabilising factor'by thrombin.
What are the indications for an emergency resuscitative
thoracotomy?
• According to the Advanced Trauma Life Support (ATLS®) guidelines,
following penetrating trauma (not blunt trauma) within 10 minutes of
pulseless electrical activity (PEA).
• It is performed by a left anterolateral thoracotomy through the 4th
intercostal space.
• Through this access, the therapeutic options include:
a) evacuation of the pericardium;
b) open cardiac massage;
c) clamping of the descending aorta (to stop distal bleeding and increase
coronary and cerebral perfusion).
Which of these is not a cause of restrictive pericarditis?

•a. tuberculosis
•b. amyloid
•c. hemochromatosis
•d. sarcoid

(A) Tuberculosis causes constrictive pericarditis.


A young man has a solitary lung lesion with calcification having
popcorn like appearance on x-ray. The most likely diagnosis is:

a) chondroma

b) chondrosarcoma

c) primary lung cancer

d) hamartoma

e) metastatic carcinoma

D The most common solitary lung lesion is the hamartoma, which presents as a calcification having popcorn like appearance on the x-ray
A 30-year-old female presents with hemoptysis. She had an unremarkable past
medical history, except for the presence of telangiectasias. A chest x-ray shows a 3.0
cm lobulated well-defined subpleural lesion in the right lower lobe. The management
should include:
a) observation
b) percutaneous needle biopsy followed by a thoracotomy if malignant
c) pulmonary angiogram followed by embolization of the feeding artery
d) segmental resection of the lesion
e) resection of the lesion with a lobectomy
C The patient has a subpleural hemangioma, it could easily be diagnosed by angiography, and then embolized as well.

A 70-year-old man has a history of cough over the past month and recent onset of
dyspnea. A chest x-ray reveals a mass in the right lower lobe. The CT guided needle
aspiration shows bronchogenic carcinoma. Indications for non resectability include
each of the following except:
a) the presence of tumor in the contralateral main-stem bronchus
b) metastatic tumor in contralateral mediastinal nodes
c) involvement of the phrenic nerve and paralysis of the diaphragm
d) invasion of the recurrent laryngeal nerve
e) invasion of the superior vena cava
A 60-year-old male presents to your office. He has a 40 year history of
smoking and was found to have a 4 cm right lower lobe mass on
routine chest x-ray. In addition there is a pleural effusion on that side.
The first thing you did was a thoracentesis of the effusion and it came
back positive for carcinoma. The next step in the management of this
patient should be:

a) video-assisted thoracostomy and biopsy of the lesion


b) bronchoscopy
c) oncology consultation for chemotherapy
d) mediastinoscopy
e) PET scan
C This patient has stage IV disease and is not a candidate for surgery. The oncology service should be consulted for further management.
Male patient 60 years old smoker (30 cig/day), mine worker
presented with chronic cough >3 weeks not respond to
medical treatment, recently coughing of blood, chest pain at
right arm, with ptosis of right eye, headache, attacks of fainting,
shortness of breath, back pain, history of loss of weight and
loss of appetite, hoarseness of voice,.
Ex.:- Cachectic, edema and swelling of right upper limb and
right side of the face, ptosis-myosis -anhydrosis of Rt eye,
By auscultation decreased air entry on RT side of chest.
What is your next step?.
• Which one of these statements is false?
• A. Lifetime cigarette smoking, quantified as ‘pack-years’, is a major risk factor for
bronchial carcinoma.
• B. Compared with non-small-cell cancer, small-cell lung cancer, formerly known as
oat cell cancer, is less common, metastasizes early and is less amenable to
surgery.
• C. Finger clubbing and hypertrophic pulmonary osteoarthropathy, sometimes
described as clinical features of lung cancer, are usually incidental findings and not
due to primary lung cancer.
• D. The appropriate treatment strategy is dependent on tumour type, tumour stage,
and the general fitness and lung function of the patient.
• E. Late survival has a direct relationship with the tumour stage at the time of
treatment.
• Which one of these statements is true?
• A. Chest radiograph (CXR) yields very useful information
about primary lung cancer.
• B. Computed tomography (CT) is only useful for guiding
fine-needle aspiration.
• C. Positron emission tomography (PET) has high
specificity for bronchial carcinoma.
• D. Sputum cytology has a high sensitivity.
• E. Invasive procedures such as mediastinoscopy,
mediastinotomy and thoracoscopy are not staging
procedures.
• Which of the following are true of FDG-PET scanning?
A. four hour fasting period is recommended before an FDG-PET study
B. F used in FDG-PET scanning has a half life of ≈110 min.
C. FDG is taken up by tumour cells and hydrolysed
D. The spatial resolution of PET is better than that of CT
E. A negative FDG-PET scan is an indication of a benign lesion
TTFFF
Raised serum glucose decreases cellular FDG uptake because both glucose and FDG compete for the same cell
surface receptor. Radionuclides used in PET scanning are typically isotopes with short half lives. FDG is avidly
taken up by tumour cells and phosphorylated. The spatial resolution of PET is about 7 mm, much lower than
that of other imaging methods, such as CT. A negative FDG-PET scan is not an absolute indication of a benign
lesion and carcinoid tumour as well as well-differentiated low negative.
Lesions greater than what size , in diameter , are considered
masses rather than nodules in the lung ? Select Best Answer
A. 20 mm B. 25 mm
C. 30 mm D. 35 mm
E. 40 mm

Which of the following radiographic characteristics makes a


pulmonary nodule less likely to be malignant ? Select Best Answer
A. Irregular boundary
B. Lobulated boundary
C. Burr- like boundary
D. Smooth margin
E. Spiculated boundary
•Why do we need to do
staging of lung cancer ?
• It helps doctors tocommunicate
among themselves
• It helps to predict theprognosis of the
patient
• It helps in deciding thetreatment plan
A 47-year-old woman is seen with a 1-wk history of progressive shortness of breath,
increasing pedal edema, weight loss, and low-grade fever. She has a 40-pack-year
smoking history. Physical examination: pulse 138 bpm; respirations 34/min; blood
pressure 100/88 mm Hg with pulsus paradoxus of 22 mm. Pertinent findings:
increased jugular venous distension, normal sinus rhythm with distant heart sounds,
and an apex beat that is difficult to palpate. Chest x-ray is shown in Figure below.
What is the most likely diagnosis?
a. Cardiac tamponade
b. Cardiomyopathy
c. Pericardial effusion without tamponade
d. Cor pulmonale
Another finding associated with this diagnosis would be
a. Kussmaul sign
b. Low voltage on ECG
c. Left ventricular hypertrophy
d. Pulmonary edema
A 65 year old male presents with fever and chest pain. Chest x ray reveals
an enlarged cardiac silhouette. ECHO reveals fluid collection around the
posterior pericardium. The next step is:
a. CT scan
b. pericardiocentesis
c. right side cardiac catheterization
d. radionuclide scan
(B) Pericardiocentesis is a safe first step to evaluate the pericardial fluid.

The pericardiocentesis reveals G-positive cocci and the nature of the fluid
is viscous. The next step is:
a. prolonged course of antibiotics
b. anterior thoracotomy
c. subxiphoid window
d. median sternotomy
(B) If a median sternotomy is done to evacuate the purulent fluid, the sternal wires have a good chance of getting infected resulting in mediastinitis. An anterior thoracotomy is preferred.
You are consulted on a 45-year-old male who is suspected of
having cardiac tamponade. Which of the following increases
response to cardiac tamponade?

A-coronary blood flow


B-mean arterial pressure
C-peripheral resistance
D-cardiac index
E-left ventricular stroke work

C During cardiac tamponade, the peripheral resistance initially increases to maintain the mean arterial pressure, since the cardiac output is falling.
•Which statement about pericardial effusions is false?

A- Pericardial volume and pressure are linearly related.


B- Uremia may be associated with both pericarditis and
effusion.
C- Tuberculous pericarditis may be calcific.
D- In cardiac tamponade filling of the heart is limited during
diastole.

(A) Acute tamponade may be caused by a very small blood clot and is not related directly to the volume. In chronic cases the pericardium can be stretched and an increase volume is tolerated. As the pressure in the
pericardial sac increases the diastolic filling pressure also increases. Compensatory mechanisms include increased sympathetic activity, increased vasoconstriction and increased heart rate. The venous pressure
rises and coronary flow is decreases with a decrease in cardiac output.
The most common cause of pericarditis is:
• a. post cardiotomy syndrome
• b. uremia
• c. idiopathic
• d. post myocardial infarction

(C) The majority of causes of pericarditis are idiopathic. Other known causes are post cardiac surgery, viruses, collagen vascular disorders and renal failure.
These patients present with general malaise, fever and chest pain. Usually conservative management is undertaken which includes NSAIDs, pain control and in
some cases even steroids for a short time.
Beck's triad is characterized by all of the following
except:
• a. elevated JVP
• b. hypotension
• c. muffled heart sounds
• d. pericardial thickening

(D) Pericardial thickening is not part of Beck's triad.


Pulsus paradox is defined as:

• a. decrease in systolic blood pressure during inspiration


• b. decrease in inspiration with a fall in blood pressure
• c. increase in systolic blood pressure with inspiration
• d. elevated JVP with inspiration

(A)Pulsus paradox is decrease in systolic blood pressure during inspiration.


Which statement is false about constrictive pericarditis?

a- ECHO is not diagnostic of constrictive pericarditis


b- Right heart catheterization will demonstrate diastolic dip
plateau pattern in right ventricular tracing
c- Pericardiectomy should be started on the right ventricle and
pulmonary artery first
d- Chest x-ray is frequently normal
(C)In constrictive pericarditis the diagnosis can be difficult.
The chest x-ray may show a normal size heart.
The x-ray may show calcification which may lead one to think about TB. Echo will
show fluid and rule out pericardial effusions and it will also reveal the heart function
but is not diagnostic of pericarditis.
The ECG will show non specific ST segment changes.
The CT can identify the thick membrane surrounding the heart but also is
not diagnostic. Cardiac catheterization is diagnostic and will show the end diastolic
pressure elevation in the right atrium pulmonary artery and left atrium.
Generally in constrictive pericarditis one observes an early rapid fall in diastolic
pressure in the right ventricle followed by a rapid rise to an elevated diastolic
plateau (square root sign). The left ventricular pressure curve has similar pattern.
The mean right atrial pressure increases with inspiration. Rapid infusion of 1000cc
of normal saline shows reversal of respiratory variation of right atrial pressure.
The pericardial resection termed decortication is usually started on the left ventricle
to prevent the patient from going into pulmonary edema and acute right heart failure
if the right ventricle is freed first.
Which of these is not a cause of restrictive pericarditis?

•a. tuberculosis
•b. amyloid
•c. hemochromatosis
•d. sarcoid

(A) Tuberculosis causes constrictive pericarditis.


An obese female has a mechanical aortic valve replacement.
Her post course is unremarkable and she is discharged home.
She returns two weeks later with complaints of lethargy,
progressive dyspnea and orthopnea. Her JVP is slightly
elevated. Besides the x ray, the next best test to diagnose her
condition is:

a. Ventilation perfusion scan


b. Right heart catheterization
c. ECHO
d. Fluoroscopy

(C) Delayed tamponade must be ruled out in any patient who presents with the above symptoms. Delayed tamponade can present anytime from 1 week to 4 months later. The chest x ray usually shows an
enlarged cardiac silhouette. ECHO will confirm the pericardial fluid accumulation. Treatment consists of pericardiocentesis or a subxiphoid window.
All of the following are useful modalities in the
assessment of penetrating cardiac injury except

A- CT scanning
B- FAST
C- pericardiocentesis
D- echocardiography
Which of the following statements about pericarditis is false?

• a. In restrictive pericarditis, filling of the heart is impaired


throughout diastole.
• b. In constrictive pericarditis, only the early diastolic filling is
diminished.
• c. Administration of fluids in restrictive pericarditis, will show
divergence of the atrial and ventricular filling curves.
• d. Administration of fluids in constrictive pericarditis will
show that both the atrial and ventricular filling curves stay
parallel.
A 57-yrs old women asthmatic, hypertensive, and metastatic breast cancer are is
brought to the emergency department because of difficulty of breathing , lethargy,
and palpitations. She had been complaining to her husband of increasing shortening
of breathing over the last several days. She recently had finished her fifth round of
chemotherapy last month and underwent radiation therapy 2 wks ago. her BP 75/40
mmhg and pulse is 12 bm. Her oxygen satiortion is 93% on room air . She is
minimally responsive to commands. Her heart sounds are faint ,but audible. She
has mild bilateral rales . And her jugular venous pressure is approximately 15 cm
H2O . A chest X-ray shows multiple lung nodules and an enlarged cardiac silhouette .
An ECG shows sinus tachycardia with low voltage QRS complex .
The most appropriate next step in management is:

A- a dilution of carbon monoxide


B- an echocardiography
C- a pulmonary angiography
D- a thoracic CT scan
A 42 yes old man is brought to the emergency department after being
stabbed in the chest with knife at a local bar, a witness says that the patients was
attacked the knife from behind and suffered multiple kicks to the abdomen. The
stab wound was with an unknown type of blade to the right chest. The Pt past
medical history is unknown and the pt allergies are unknown, the Pt last ate three
hours ago. On initial presentation ,there are large patches of dried blood on his shirt
and face and his lips are covered with dried blood. He is diaphoretic but speaking
in full sentences . He has multiple stab wounds on his right chest, both inferior and
superior to the right nipple. A chest tube is inserted on the right side with drainage
of 1200cc of blood. A diagnostic peritoneal lavage is negative for any blood . His
BP is 90/50 mmhg,. Physical examination shows jugular venous pulsation visible at
14 cm H2O . The heart sounds are distant and barely audible. The lung are clear
bilaterally.
What is the most likely cause for this patients hypotension?
A) myocardial contusion
B) pericardial tamponade
C) pericarditis
D) pulmonary contusion
In a course of robbery, a young woman is stabbed repeatedly. On arrival at the
emergency department, she is shivering and asks for blanket and a drink of
water, she is noted to be pale and perspiring. Her BP is 72/50 mm hg and her
pulse is 130 bpm. Her neck and forehead veins are distended. A quick initial
survey reveals entry wound in her left chest and upper abdomen. She has
bilateral breath sounds and a scaphoid, nontender abdomen. As IV infusion of
Ringers lactate are started, her systolic BP drops further to 40 mm hg. No
distal pulses can be felt, and she loses consciousness. Her central venous
pressure at that time is 28 cm H2O. Which of the following is the most
appropriate next step in management?

• A) chest X-ray to direct further therapy


• B) bilateral chest tube
• C) diagnostic peritoneal lavage
• D) evacuation of the pericardial sac
What is the best step in management for an 80 year
old male with adult-onset asthma and worsening
dyspnea on exertion wheezing ?
Select Best Answer
A.Observation
B.PET scan
C.Radiation Therapy
D.Bronchoscopy
Quiz
Case 1
• A previously well 23 year old man is brought to your
Emergency Department acutely short of breath
after developing left sided chest pain at work.
• On arrival, he appeared pale and sweaty and was
hypotensive.
• A CXR was taken immediately after a procedure
was performed to stabilise his condition. His vital
signs are now normal.
- Describe and interpret his CXR
- Outline your treatment options
Answer
• Chest X-ray showing a pneumothorax
• Needle thoracostomy catheter in situ
• No evidence of radiological tension
Case 2
• A 57 year old female car driver presents following a head on
collision with a bus at 60Kph.
• Her observations are listed:
HR98, BP130/90mmHg, Resp 24, SpO2 98%

- Describe and interpret her X-ray


- Outline your management options
Answer(s)
• Widened Mediastinum
• Clavicle Fracture
• Rib Fractures
• ?Right Haemopneumothorax
Case 3
• An 80 year old male pedestrian is brought to your emergency
department 30 minutes after being struck by a motorcycle at
high speed.

• What does the X ray show (Use DRABCDE approach)


Answer(s)
• Chest X-ray of trauma patient showing multiple rib fractures and
underlying area of pulmonary contusion or haemothorax
Case 4
• A 20 year old man presents to your emergency department with
central chest pain that commenced after recreational drug use at
a party two hours earlier.
• His observations are:
• Describe and interpret his Chest X-ray
• Outline your management.
Answer(s)
• Mediastinal emphysema, Subcutaneous emphysema.
• Pneumothorax may be due to attempted subclavian or jugular vein
puncture in IV drug users, rupture of drug-related bullae or rarely
rupture of peripheral pulmonary abscesses
• The large airway pressure changes involved in inhalational
manoeuvres employed in crack or cannabis use may also lead to
rupture of distal airways. Air may then track into the pleural space
or mediastinum, manifesting as pneumothorax or
pneumomediastinum Roszler
( MH et al)
Case 5
• A 52 year old woman presents to your emergency department with
gradually increasing breathlessness over the preceding three days.
It is one week since her last chemotherapy treatment for cancer.
• Her observations are:
• Describe the CXR findings
• Outline your Differential Diagnosis
Answer(s)
• X-ray showed
• Large left pleural effusion
• Multiple discrete lung parenchymal lesions typical of metastatic lung
disease
• Porta-cath

• OTHER:
• ?Mastectomy ?O2 Mask
Pleural Effusions
• According to Light's criteria a pleural effusion is exudative if at
least one of the following exists:
• The ratio of pleural fluid protein to serum
protein is greater than 0.5
• The ratio of pleural fluid LDH and serum LDH
is greater than 0.6
• Pleural fluid LDH is greater than 0.6 times the
normal upper limit for serum. (i.e 0.6 of 200)
Pleural Effusions - Causes

•Transudates
• Congestive Heart Fauklrue, lvier Fauirleu, Renal Faiulre, Nephrotic
syndrome, Hypoalbuminaemia, Enteropathy, Dialysis

•Exudates
• Lung Ca, TB, Infections (Bacterial), RA, Pancreatitis, Subphrenic
Abscess, Meig’s Syndrome, Dressler’s Syndrome, SLE, Lymphoma,
Hypothyroid, PE, Mesothelioma, Yellow Nail Syndrome, Vasculitis
Case 6
• A distressed 60 year old man from a nursing home is brought
into the ED having ‘choked on his dentures’
• His CXR is shown

- Describe the CXR findings


- What further investigations may be indicated in this man?
Answers
• A CXR showing dentures overlying the mediastinum
• Probable air in the neck soft tissues.
• Possible oesophageal perforation due to a foreign body.
Case 7
• An 18 year old woman with a history of asthma since childhood
presents with a one month history of weight loss, cough and
malaise
• She has been treated with two courses of antibiotics by her
local GP. She now presents with increasing shortness of breath.
- Describe the X-ray
- List your differential diagnosis
Answers
• Patchy Opacification
• ?Fluid in Pleural Space
• Broad Differential Diagnosis
• ?Pneumonia
Case 8
• A 35 year old homeless man presents with two months of
increasing cough. He has no other medical past history.
- Describe his Chest X-ray
What are the possible causes of this appearance?
Lung Cavity
• Chest X-ray showing large cavitating lesion in right hemithorax with soft
tissue density in lowerzone laterally.
• Causes:
• Infective cause including TB, Fungi, Aspiration
• Malignancy
• Abscess and other bacterial infection Staph/
Kleb
• Wegner’s and Massive Fibrosis
• PE
Case 9
• A 4 year old boy presents to the emergency department
following a choking episode at home 30 minutes previously.
- Describe and interpret his X-ray
- What factors would determine further management?
Answer
• CXR showing round radio-opaque midline foreign body.
• Round metallic FB in lower oesophagus
• Most likely coin but need to consider button battery

• NB: The classic teaching is that on an AP/PA radiograph coins in


the esophagus are oriented in the coronal plane whereas coins
in the trachea are oriented sagittally
Button Battery Ingestion
• Almost Exclusivly in the paediatric Population
• Majority (<15mm) pass throught the GIT uneventfully
• Larger Batteries (>20mm) may Lodge in the Oesophagus
• This leads to Significant Complications
• This is due to direct pressure and release of alkali
• Mercury may also be ‘released’ from the battery
Button Batteries
• A button battery in the Oesophagus requires removal ideally
within 6 hours
• This also allows examination of the mucosa
• If beyond the oesophagus it may pass naturally
• ENT batteries should be removed urgently
• Confusion may arise between coins and batteries – the battery
has a stepped appearance
• Follow up with repeat X-rays at 48 hours
Case 10
• You are called by the nurse on the ward to check position of NG
tube

• Where is the NGT?


• Where should it be?
A 34 year old is shot in the chest. ECHO reveals that the bullet is lodged
in the atrial septum. Two days later the patient develops a syncopal
attack and CT scan shows that the bullet is now at the right carotid
bifurcation.
The best approach to this patient is:
a. to remove the bullet from the carotid via a neck incision
b. to open both the chest and neck
c. to do a percutaneous removal of bullet via the femoral artery in the
angio lab
d. to wait and remove the bullet from the carotid at a later time

B)This patient has a paradoxical embolus of a bullet via an atrial septal defect. At this time both the atrial septal defect has to be closed and the bullet must be removed from the neck. The only way the bullet
could have gotten from the right atrium to the right carotid bifurcation is via the atrial septal defect. Both the septal defect and the bullet have to be treated.
A case of postpneumonectomy Rt side empyema with BPF
coughing of huge amount of purulent sputum related to
posture, how you can manage?
Poor surgically candidate patient with recurrent BPF after
surgical treatment of SSP with prolonged air leakage,
how you can manage this patient?
Which of the following regarding the blood supply to pedicled
tissue flaps used in chest wall reconstructions are correct?
A. The blood supply of the rectus abdominis muscle flap is the superior
epigastric artery
B. The blood supply of the serratus anterior muscle flap is the lateral
thoracic artery
C. The blood supply of the pectoralis major muscle flap is the
thoracoacromial artery, when used as a turn over flap
D. The blood supply of the latissimus dorsi muscle flap is the
thoracodorsal artery
E. The blood supply of the omental flap is based on either the right or left
gastroepiploic arteries

TTFTT The pectoralis can be used as a turnover flap based on the medial sternal perforators or rotated into the sternal wound based on the thoracoacromial system.
Which of the following statements are true?
A. Heparin inhibits anti-thrombin IIl
B. Bivalirudin inhibits the catalytic activity of thrombin
C. Ancrod acts by enzymatically digesting thrombin
D. Warfarin inhibits synthesis of clotting factors II/VII/IX/X
E. Low molecular weight heparin inhibits factor Xa

FTFTT
Heparin enhances anti-thrombin IIl. Ancrod acts by enzymatically digesting fibrinogen.
The aortic opening in the diaphragm transmits the
following
A.The thoracic duct
B.The vagus nerves
C.The phrenic nerves
D.The inferior vena cava
E.The superior epigastric arteries

TFFFF

It is located approximately at the level of the twelfth thoracic vertebra and transmits the aorta and thoracic duct and often the the azygous vein.
Which of the following are correct regarding the trachea?
A. The adult trachea is 20 cm from the cricoid cartilage to the
carina
B. It consisits of 18-22 cartilaginous rings
C. Its mucosa is of a ciliated-columnar type
D. The blood supply to the trachea travels in lateral tissue pedicles
E. The muscular tissue consists of two layers of striated muscle

FTTTF
The adult trachea is 11 cm long at approximately two rings/cm of length. During tracheal mobilisation preserving the lateral blood supply and a tension free
anastomosis is paramount to successful outcome.The muscular tissue consists of two layers of non-striated muscle,an outer longitudinal and inner transverse
(Trachealis muscle) that forms a thin layer which extends transversely between the ends of the cartilages.
Which of the following are true regarding the phrenic
nerves?
A. They arise from the dorsal segments of the 3rd to 5th cervical
dorsal rami
B. They cross the second part of the subclavian arteries bilaterally
C. They enter the thorax between the subclavian artery and vein
D. The left phrenic nerve pierces the central tendon to supply the
left hemidaiphragm
E. The accessory phrenic nerve joins the main nerve at the root of
the neck
The images are of a 17-year-old woman with Marfan
syndrome. She had moderate preoperative aortic valve
regurgitation.
The optimal surgical procedure for this patient is
• A aortic root replacement with a pulmonary autograft
• B aortic root replacement with a valved composite graft
• C aortic valvuloplasty and ascending aorta replacement
• D valve-sparing aortic root replacement

. A conservative and safe approach is the Bentall procedure, which involves replacing the aortic root and valve with
a valved composite graft and coronary artery button reimplantation
A case of postpneumonectomy Rt side empyema with BPF
coughing of huge amount of purulent sputum related to
posture, how you can manage?
Recurrent BPF after surgical treatment of Pt with SSP with
prolonged air leakage, how you can manage?
Q22 Which of the following statements regarding the embryology of the heart and great vessels are
correct?
A.The ductus arteriosus is derived from the fourth branchial arch artery
B.The distal portion of the aortic arch is derived from the sixth branchial arch artery
C.The right recurrent laryngeal nerve lies in relation to the distal portion of the right sixth branchial
arch artery
D.The bracheocephalic artery is derived from the right horn of the aortic sac
E.The left subclavian artery is formed from the left seventh cervical intersegmental artery

FFFTT
The embryo has two dorsal aortae that communicate with an aortic sac via several pairs of
branchial arch arteries. The right dorsal aorta largely and the first,second and fifth pairs of arch
arteries involute.The third pair of arch arteries forms the common and proximal internal carotid
arteries. The left fourth arch artery becomes part of the aortic arch while the right fourth arch
artery becomes the root of the right subclavian artery. The left subclavian artery is derived from an
intersegmental artery arising from the dorsal aorta. The brachiocephalic trunk arises from the
aortic sac.(Refer to The circulatory system in:Moore KL and Persaud TVN.The developing human:
clinically orientated embryology,Sixth Edn.WB Saunders Company,1998 for more detail.)
Q56 Which of the following regarding the blood supply to pedidied tissue
flaps used in chest wall reconstructions are correct?
CA.The blood supply of the rectus abdominis muscle flap is the superior
epigastric artery
CB.The blood supply of the serratus anterior muscle flap is the lateral
thoracic artery
C.The blood supply of the pectoralis major muscle flap is the
thoracoacromial artery,when used as a turn over flap
D. The blood supply of the latissimus dorsi muscle flap is the thoracodorsal
artery
E.The blood supply of the omental flap is based on eiter the right or left
gastroepiploic arteries
TTFTT
The pectoralis can be used as a turnover flap based on the medial sternal
perforators or rotated into the sternal wound based on the thoracoacromial
system.
Q53 Which of the following are true of FDG-PET scanning?
CA.A four hour fasting period is recommended before an FDG-PET
study
B. F used in FDG-PET scanning has a half life of ≈110 min.
C.FDG is taken up by tumour cells and hydrolysed
D.The spatial resolution of PET is better than that of CT
E. A negative FDG-PET scan is an indication of a benign lesion
TTFFF
Raised serum glucose decreases cellular FDG uptake because both glucose and FDG
compete for the same cell surface receptor.Radionuclides used in PET scanning are
typically isotopes with short half lives. FDG is avidly taken up by tumour cells and
phosphorylated. The spatial resolution of PET is about 7 mm,much lower than that of
other imaging methods, such as CT. A negative FDG-PET scan is not an absolute
indication of a benign leion and carcinoid tumours as well as well-differentiated low
Q50 Differences between the right and left main stem bronchi include
A.The right main stem bronchus is supplied by one bronchial artery
B.The left main stem bronchus is supplied by two bronchial arteries
C.The right bronchial artery arises directly from the aorta
D.The right bronchus is narrower than the left
E.The right main stem bronchus enters its lung lower than the left enters
its lung

TTFFF
The right mainstem bronchus is thus more susceptible to the proximal descending
aorta.
Q19 Which of the following statements are true?
A. Heparin inhibits anti-thrombin IIl
B. Bivalirudin inhibits the catalytic activity of thrombin
C. Ancrod acts by enzymatically digesting thrombin
D. Warfarin inhibits synthesis of clotting factors II/VII/IX/X
E. Low molecular weight heparin inhibits factor Xa
FTFTT
Heparin enhances anti-thrombin IIl. Ancrod acts by enzymatically digesting
fibrinogen.
Q6 Which of the following are true regarding Heparin?
A. It is a carbohydrate that consists of a variably sulphated repeating disaccharide
B. It is a naturally occurring anticoagulant produced by basophils and mast cells
C. It works by inactivating anti-thrombin III
D. It plays an important role in the breakdown of clot
E. It results in the formation of soft clot
TTFFT
Heparin is a member of the glycosaminoglycan family of carbohydrates. It acts as
an anticoagulant, preventing the formation of clot and the extension of existing
clot. Heparin binds to the enzyme inhibitor antithrombin III resulting in its active
site being exposed. The activated form then inactivates thrombin and other
proteases involved in blood clotting, such as Factor Xa. Although it is not a
thrombolytic, heparin prevents the formation of a stable fibrin clot by inhibiting the
activation of the 'fibrin stabilising factor'by thrombin.
Q10 Which statements are true regarding the activated clotting time?
A. It is used when heparin levels are too high to allow monitoring with an APTT
or when a rapid result is necessary to monitor treatment
B. The ACT has good sensitivity and range with high doses of heparin,but its
sensitivity is significantly diminished at lower levels
C. The ACT is measured in international units
D. The ACT may be influenced by a patient's platelet count and platelet function
E. The temperature of the blood my also affect ACT results
TTFTT
The APTT test involves an in-vitro clotting reaction and at high levels of heparin
it will not clot. It is measred in seconds.
Q17 The following are true regarding warfarin
A.It enhances vitamine K epoxide reductase
B.It enhances the activity of protein C and protein S
C. It decreases the synthesis of vitamin K dependent cotting factors II,VII,IX and
X
D.African americans are relatively resistant to warfarin, while asian americans
are more sensitive to its effects
E.Skin necrosis occurs more frequently in women and in patients with pre-
existing protein C deficiency
FFFTT
Warfarin inhibits epoxide reductase diminishing available vitamin K and Vitamin
K hydroquinone, which inhibits the carboxylation of the coagulation factors at
glutamic acid residues, making them incapable of binding to the endothelial
surface of blood vessels.The coagulation factors are produced, but have
decreased functionality due to undercarboxylation. Warfarin activity is
determined partially by genetic factors. Skin necrosis is less common in men
Q74 Which of the following is correct regarding reactions to protamine?
A.A type I reaction is hypotension
B. A type IIA reaction is an anaphylactic reaction
C.A type IIB reaction is an anphylactoid reaction
D.A type III reaction is catastrophic pulmonary vasoconstriction
E. Administration of protamine directly into the left side of the heart
reduces the cardiovascular effects of protamine administration
TTTTT
Q49 Which are true of the papillary muscles and their chordae?
A. The anterior papillary muscle of the right ventricle inserts into anterior and
posterior tricuspid valve leaflets
B. The anterior papillary muscle of the left ventricle is larger than the posterior
one
C. The mitral valve has basal type chordae only associated with the anterior
leaflet
D. Papillary muscle rupture of the mitral valve involves the anterior papillary
muscle more often than the posterior
E. Papillary muscle rupture occurs with a relatively small infarction in half the
cases
TTFFT
Mitral valve basal chordae are only associated with the posterior leaflet,The
posterior papillary muscle is involved in about 75% of cases and the anterior in
Q44 The aortic opening in the diaphragm transmits the following
A.The thoracic duct
B.The vagus nerves
C.The phrenic nerves
D.The inferior vena cava
E.The superior epigastric arteries
TFFFF
It is located approximately at the level of the twelfth thoracic vertebra and
transmits the aorta and thoracic duct and often the the azygous vein.
Q45 Which of the following are correct regarding the trachea?
A.The adult trachea is 20 cm from the cricoid cartilage to the carina
CB.It consisits of 18-22 cartilaginous rings
C.Its mucosa is of a ciliated-columnar type
D.The blood supply to the trachea travels in lateral tissue pedicles
E.The muscular tissue consists of two layers of striated muscle

FTTTF
The adult trachea is 11 cm long at approximately two rings/cm of length. During
tracheal mobilisation preserving the lateral blood supply and a tension free
anastomosis is paramount to successful outcome.The muscular tissue consists
of two layers of non-striated muscle,an outer longitudinal and inner transverse
(Trachealis muscle) that forms a thin layer which extends transversely between
the ends of the cartilages.
Q40 Which of the following statements regarding functional
differences between venous conduits and arterial conduits are correct?
A. Veins have a poorly developed internal elastic lamina
B. Heparan sulphate is more expressed in veins than in arteries
C. Production of nitric oxide is lower in veins than in arteries
D. Production of prostacyclin is highe in veins than in arteries
E. Thrombin vasoconstricts saphenous vein whilst dilating internal
mammary arteries
TFTFT
Q33 Which of the following are true regarding the phrenic nerves?
A.They arise from the dorsal segments of the 3rd to 5th cervical dorsal rami
B.They cross the second part of the subclavian arteries bilaterally
C.They enter the thorax between the subclavian artery and vein
D.The left phrenic nerve pierces the central tendon to supply the left
hemidaiphragm
cE.The accessory phrenic nerve joins the main nerve at the root of the neck
FFTFT
The left phrenic nerve crosses the first part of the subclavian artery.The left phrenic
nerve piercesthe dome of the left hemidiaphragm anterior to the centra tendon.
Q16 Which of the following statements regarding the anatomy of the tricuspid
valve are correct?
A.The normal tricuspid valve usually has three leaflets and three papillary
muscles
B.Congenital apical displacement of the tricuspid valve is called Ebstein's
anomaly and typically causes significant tricuspid regurgitation
C.The orifice is smaller than the mitral orifice and is circular
D.The septal leaflet is the largest leaflet
E. The membranous septum and contained A-V conduction system lies
beneath the septal leaflet
TTFFT
Ebstein's anomaly is a congenital malformation of the heart characterised by
apical displacement of the septal and posterior tricuspid valve leaflets,leading to
atrialisation of the right ventricle with a variable degree of malformation and
displacement of he
Q14 Regarding the central venous pressure
A. It measures the right atrial pressure
B. It relates directly to the right ventricular afterload
C. In a ventilated patient on PEEP the true CVP is obtained by subtracting the
PEEP from the CVP reading
D. During the 'v'wave the tricuspid valveis open
E. Large 'a'waves may occur in tricuspid regurgitation
TFFFF
Subtracting the PEEP from the CVP reading tends to underestimate the actual
value.On the other hand excessive PEEP can severely reduce the CVP. Large 'a'
waves occur in tricuspid stenosis,pulmonary stenosis and pulmonary
hypertension. Cannon 'a'waves occur when the right atrium contracts against a
closed tricuspid valve as occurs in complete heart block.
The images are of a 17-year-old woman with Marfan syndrome. She had
moderate preoperative aortic valve regurgitation.
The optimal surgical procedure for this patient is
• A aortic root replacement with a pulmonary autograft
• B aortic root replacement with a valved composite graft
• C aortic valvuloplasty and ascending aorta replacement
• D valve-sparing aortic root replacement

. A conservative and safe approach is the Bentall procedure, which involves


replacing the aortic root and valve with a valved composite graft and coronary artery
button reimplantation
The coronary artery anatomy most likely to be associated with sudden
death in a previously asymptomatic adolescent is
• A left anterior descending coronary artery branching from the right
coronary artery
• B origin of the left coronary artery from right aortic sinus of Valsalva
• C origin of the right coronary artery from left aortic sinus of Valsalva
• D right coronary artery-to-right ventricle fistula
Coronary artery anomalies are the second leading cause of sudden death in older children and young
adults. In particular, anomalous aortic origin of a coronary artery (AAOCA) from the opposite sinus of
Valsalva with an intramural course is the primary coronary cause of sudden death in previously
asymptomatic adolescents. In pathology series the predominant anatomy in sudden death patients is
origin of the left coronary artery from the right sinus (see pictures). This configuration is considerably
more rare than origin of the right coronary from the left sinus. Most practitioners agree that surgical
intervention is indicated if a patient presents with signs and/or symptoms that suggest myocardial
ischemia. It is more common to recommend operative repair if this is an anomalous left coronary
artery than an anomalous right coronary artery. There is some evidence that a longer intramural
coronary length correlates with symptoms. The most common surgical intervention for AAOCA from
the adjacent sinus of Valsalva is coronary artery unroofing (see illustration). The Congenital Heart
Surgeons’ Society has developed a prospective registry to evaluate the outcome of surgical
intervention in these patients.

Origin of the left anterior descending coronary artery from the right coronary artery (single aortic orifice)
is present in 5%-8% of patients with tetralogy of Fallot (TOF). This coronary pattern is usually not
obstructed and has not been associated with sudden death. However, intraoperative injury during
repair of TOF can be lethal. A coronary artery-to-right ventricle fistula causes a left-to-right shunt, but
this has not been associated with myocardial ischemia.
A 20-year-old otherwise healthy man presented with chest pain associated with
vigorous exercise. He recently had an episode of syncope while playing
basketball. Initial transthoracic echocardiography demonstrated anomalous
aortic origin of the right coronary artery from the left sinus of Valsalva. A
subsequent CT angiogram confirmed the presence of a right coronary artery
arising from the left sinus, with an intramural course through the wall of the
aorta, traveling between the aorta and the pulmonary artery.

Which of the following is the best option for treatment?

• A coronary artery bypass grafting


• B coronary reimplantation
• C percutaneous stenting
• D unroofing the artery
The end
than you
k

You might also like